NCERT Solutions for Class 10 Geography Chapter 1 Resources and Development

Dipen

  • 15th January 2024

NCERT Solutions for Class 10 Social Science Geography Chapter 1 Resources and Development help students to score good marks in the exams. These NCERT Solutions are prepared by expert teachers and based on the latest pattern and edition of NCERT book. Here we have provided answers to all the questions in a very easy language.

Class 10 Geography Chapter 1 Resources and Development Questions and Answers

Question 1. Multiple choice questions 

(i) Which one of the following is the main cause of land degradation in Punjab?

(a) Intensive cultivation (b) Deforestation                (c) Over irrigation (d) Overgrazing

Answer: (c) Over irrigation

(ii) In which one of the following states is terrace cultivation practised?

(a) Punjab    (b) Plains of Uttar Pradesh                                     (c) Haryana (d) Uttarakhand

Answer: (d) Uttarakhand 

(iii) In which of the following states is black soil found?

(a) Jammu and Kashmir       (b) Gujarat    (c) Rajasthan           (d) Jharkhand

Answer: (b) Gujarat 

Question 2. Answer the following questions in about 30 words. 

(i) Name three states having black soil and the crop which is mainly grown in it. 

(ii) What type of soil is found in the river deltas of the eastern coast? Give three main features of this type of soil. 

(iii) What steps can be taken to control soil erosion in the hilly areas? 

  Answer: (i) Maharashtra, Madhya Pradesh and Chhattisgarh are states having black soil. Cotton is mainly grown in black soil. 

(ii) Alluvial Soil is found in the river deltas of the eastern coast.

Three features of alluvial soil:

  • Alluvial soil is rich in nutrients like potash and lime, making it highly fertile.
  • Alluvial soil has a fine texture, owing to the fine silt deposited by river waters. The fine particles allow the soil to retain moisture effectively.
  • The fertility and texture of Alluvial soil make it ideal for agriculture. They are ideal for growing sugarcane, wheat and paddy.

(iii) In hilly areas, soil erosion can be controlled by ploughing across contour-lines, making use of terrace farming techniques and using strips of grasses to check soil erosion by wind and water. 

Question 3. Answer the following questions in about 120 words. 

(i) Explain land use pattern in India and why has the land under forest not increased much since 1960-61? 

(ii) How have technical and economic development led to more consumption of resources? 

Answer: (I) The land use pattern in India involves diverse uses, including agriculture, forestry, mining, construction, and other activities. Since 1960-61, the land under forest in India has not increased significantly due to several factors:

  • The growing population has led to increased demand for food, resulting in the conversion of forest land to agricultural land.
  • Rapid urbanization and industrialization have required more land, often at the expense of forest areas.
  • Over the years, forests have been overexploited for timber, fuelwood, and other resources, leading to deforestation.
  • While there have been policies aimed at increasing forest cover, their implementation has often been ineffective or insufficient.

(ii) Technical and economic development has led to more consumption of resources on account of various factors such as:

  • Technological advancements have enabled mass production in industries, increasing the consumption of natural resources like minerals, fossil fuels, and water.
  • Economic development has fueled urbanization, leading to the construction of more buildings, roads, and infrastructure, thereby consuming large amounts of materials like cement, steel, and energy.
  • Technological advancements in transportation have resulted in more vehicles and travel, increasing the consumption of fossil fuels.

More study materials for CBSE Class 10

CBSE Notes for Class 10
CBSE Sample Papers for Class 10Important Questions for Class 10
RS Aggarwal Solutions For Class 10RD Sharma Solutions For Class 10

Leave a Reply Cancel Reply

Your email address will not be published. Required fields are marked *

Name  *

Email  *

Add Comment  *

Post Comment

close menu

NCERT Solutions for Class 10 Geography Chapter 1 Resources and Development

assignment 1 geography class 10

NCERT Solutions for Class 10 Geography Chapter 1 Resources and Development in English and Hindi Medium updated for new academic session 2024-25 based on current CBSE Curriculum and new NCERT Books for 2024-25. Ask your doubts and reply to the questions asked by other users through Discussion Forum of Tiwari Academy. Download Offline Apps for offline use without internet it works properly.

NCERT Solutions for Class 10 Geography Chapter 1

  • Class 10 Geography Chapter 1 NCERT Solutions
  • Class 10 Geography Chapter 1 Hindi Medium
  • Class 10 Geography Chapter 1 Important Questions
  • NCERT Book Geography Chapter 1
  • Notes and Study Material Chapter 1
  • Class 10 Geography NCERT Solutions

Class:10
Subject:Social Science – Geography
Chapter 1:Resources and Development
Session:2024-25

NCERT Solutions for Class 10 Geography Chapter 1 in PDF format updated for new academic session 2024-25 is given here to free download. Download NCERT Solutions based Offline Apps for offline use. Contents are updated as per new NCERT Books following the latest CBSE Syllabus 2024-25.

Exatra Questions Class 10 Geography Chapter 1

What is meant by resources.

Everything available in our environment which can be used to satisfy our needs, provided, it is technologically accessible, economically feasible and culturally acceptable can be termed as ‘Resources’.

What are Biotic Resources?

Resources which are obtained from biosphere and have life known as Biotic Resources. Such as human beings, flora and fauna.

What are Abiotic Resources?

All those things which are composed of non-living things are called Abiotic Resources. For example: Rocks and metals.

What are Renewable resources?

The Resources which can be renewed or reproduced by physical, chemical or mechanical processes are known as renewable resources. For example: Solar and Wind

What is meant by Non Renewable Resources?

These resources take millions of years in their formation. Some of the resources like metals are recyclable and some like fossil fuels cannot be recycled and get exhausted with their use.

What is Alluvial Soil?

Made of river deposits, consist of sand silt and clay, its types-Bangar and Khadar. Intensively cultivated, supports dense population.. Ideal for paddy, wheat, sugarcane. Northern Plains and deltas.

It includes human activities like deforestation, over grazing, construction, mining, defective method of farming etc. Natural forces like wind, glacier and water flow.

Development should take place without damaging the environment. Present development status should continue and grow in future. Development in the present should not compromise with the needs of the future.

An equitable distribution of resources has become essential for a sustained quality of life and global peace. If the present trend of resource depletion by a few individuals and countries continues the future of our planet is in danger. Resource planning is essential for sustainable existence of all forms of life. Utilizing the resources in a judicial manner so as our future generation is not deprived of them.

The running water cuts through the clayey soils and makes deep channels known as gullies. This makes the land bad land and in the Chambal basin such land is known as ravines.

Important Questions on Class 10 Geography Chapter 1

Name three states having black soil and the crop which is mainly grown in it..

Maharashtra, Madhya Pradesh and Chhattisgarh are some states where black soil is found and cotton is the main crop grown on black soil.

What type of soil is found in the river deltas of the eastern coast? Give three main features of this type of soil.

Alluvial soil is found in the river deltas of the eastern coast. Alluvial soil is rich in potash, phosphoric acid and lime. It has a high water retention capacity and it is highly fertile soil.

What steps can be taken to control soil erosion in the hilly areas?

Terrace farming and shelter belt plantation can be done to prevent soil erosion in hilly areas.

What are the biotic and abiotic resources? Give some examples.

Biotic Resources: All living organisms in our environment are called biotic resources. For example, trees, animals, insects, etc. Abiotic Resources: All non-living things present in our environment are termed as abiotic resources. For example – earth, air, water, metals, rocks, etc.

Explain land use pattern in India and why has the land under forest not increased much since 1960-61?

About 45% of land is used as net sown area, i.e. for farming. About 22% of the land is under forest and the rest of the land is used for various purposes; like housing, recreation and industrial activities. Increasing population and subsequent increase in demand for resources is the main reason that forested land has not increased much during this period.

How have technical and economic development led to more consumption of resources?

Economic development creates demand for various resources and technical development gives the knowhow to exploit those resources. Thus, technical and economic development; together lead to more consumption of resources.

NCERT Solutions for Class 10 Geography Chapter 1

Forest and Wildlife Resources »

Copyright 2024 by Tiwari Academy | A step towards Free Education

google-news

Talk to our experts

1800-120-456-456

NCERT Solutions for Class 10 Geography Chapter 1 Resources and Development

  • NCERT Solutions
  • Social Science Contemporary India
  • Chapter 1 Resources And Development

ffImage

NCERT Solutions for Chapter 1 Resources and Development Class 10 - FREE PDF Download

Vedantu's Class 10 Geography NCERT Solutions are your life raft! Aligned with the latest CBSE Class 10 Social Science syllabus , the Class 10 Geography Chapter 1 offers a comprehensive guide to understanding this pivotal era. Dive deep into sustainable development, resource planning, land resources, and more, explaining clearly and breaking down complex terms, making geography accessible and engaging.

toc-symbol

Conquer your exams with a solid grasp of Chapter 1 content, boost your historical knowledge, and develop critical thinking skills. Download your FREE PDF for resources and development class 10 today and embark on a successful journey. 

Glance on NCERT Solutions for Class 10 Geography Chapter 1 Resources and Development

Uncover the different types of resources – natural, human, and man-made – and understand how we classify them based on their availability (renewable vs. non-renewable) with class 10th geography chapter 1.

Learn how resource utilisation is linked to a country's economic and social progress. 

Explore the concept of sustainable development and its importance for the resources and development class 10 questions and answers.

Gain insights into the challenges of resource depletion and environmental degradation. 

Discover strategies for responsible resource management and conservation practices and can also refer to class 10 geography chapter 1 question answer PDF download.

See how resource availability from the Resources and Development class 10 PDF is impacted by land degradation, soil erosion, and deforestation.

Analyse case studies to understand the interconnectedness of resources and development from Geography Chapter 1, class 10.

Related Chapters

Arrow-right

Access NCERT Solutions for Class 10 Geography Chapter 1 - Resources and Development

1. Multiple choice questions. 

(i) Which one of the following is the main cause of land degradation in Punjab? 

(a) Intensive cultivation 

(b) Deforestation

(c) Over irrigation 

(d) Overgrazing

Ans: (c) Over irrigation

(ii) In which one of the following states is terrace cultivation practiced? 

(a) Punjab 

(b) Plains of Uttar Pradesh

(c) Haryana

(d) Uttarakhand

Ans:  (d) Uttarakhand

(iii) In which of the following states black soil is predominantly found? 

(a) Uttar Pradesh

(b) Maharashtra 

(c) Rajasthan

(d) Jharkhand

Ans: (b) Maharashtra 

2. Answer the following questions in about 30 words. 

(i) Name three states having black soil and the crop which is mainly grown in it. 

Ans: Maharashtra: This state is known for its black cotton soil, particularly suitable for growing cotton.

Madhya Pradesh: Similar to Maharashtra, Madhya Pradesh also has black soil regions where cotton is a major crop.

Andhra Pradesh: Cotton cultivation thrives in Andhra Pradesh's black soil areas. While cotton is prominent, other crops like tobacco can also be grown in this type of soil.

(ii) What type of soil is found in the river deltas of the eastern coast? Give three main features of this type of soil. 

Ans: The type of soil found in the river deltas of the eastern coast of India is alluvial soil. Here are three main features of alluvial soil:

High fertility: Alluvial soil is rich in nutrients like potash, phosphoric acid, and lime due to the constant deposition of sediments from rivers. This makes it highly fertile and ideal for agriculture.

Light and well-drained: Alluvial soil is generally loose and sandy in texture, allowing for good drainage and aeration. This is beneficial for plant root growth and prevents waterlogging.

Varied composition: The exact composition of alluvial soil can vary depending on the source river and the distance from the delta.

(iii) What steps can be taken to control soil erosion in the hilly areas?

Ans: Land Management Practices:

Contour ploughing and planting

Cover cropping

Drainage ditches

Rotational grazing

Bioengineering

Education and Awareness:

3. Answer the following questions in about 120 words.

(i) Explain land use patterns in India and why has the land under forest not increased much since 1960-61? 

Ans: India's land use reflects a balancing act. Over 50% is dedicated to agriculture, with forests covering about 22% (below the recommended 33%). 

Urbanization and infrastructure needs are also rising. 

Limited forest cover increase since the 1960s stems from competing needs for land, encroachment, deforestation, and slow forest regrowth. 

Sustainable practices like agroforestry and stricter environmental regulations are key to achieving a more balanced and sustainable land use pattern in India.

(ii) How have technical and economic development led to more consumption of resources?

Ans: Technical and economic advancements have become double-edged swords for resource consumption. 

Improved technology allows for greater production, requiring more materials. 

New industries and products constantly emerge, each demanding resources. 

As economies grow, living standards rise, leading to increased consumption of goods and services. 

While technology might unlock new resources, it can also accelerate their exploitation. 

To ensure a sustainable future, we need to find a balance between development and responsible resource management through methods like renewable energy and promoting conservation.

Topics Covered in Class 10 Geography Chapter 1 

Geography Class 10 Chapter 1 Topics and Subtopics

 

Benefits of NCERT Solutions For Geography Class 10 Chapter 1

A few advantages you will get by opting for the class 10 geography chapter 1 question answers are:

Gain a clear understanding of different resource types (natural, human, man-made) and their classification with geography class 10 chapter 1 question answers.

Enhance your exam skills with diverse question formats aligned with the CBSE curriculum for Geography class 10, chapter 1.

Break down complex concepts with easy-to-follow explanations, ensuring a solid grasp of the material for Geography Chapter 1 class 10.

Develop critical thinking skills by analysing real-world examples like land degradation and deforestation by referring to resources and development class 10 PDF.

Learn responsible resource management and conservation strategies with resources and development class 10 questions and answers.

Download your FREE PDF of NCERT Solutions for class 10th Geography Chapter 1 today and become a champion of sustainable development!

In addition to these NCERT Solutions, the official website provides Class 10 Resources and Development Revision Notes . Utilise both resources for effective practice and a deeper understanding of the chapter.

Mastering resources and development is key to a sustainable future. NCERT Solutions for Class 10 Geography (Chapter 1) empowers you. Make informed choices about resource use, advocate for sustainable practices, and become an active citizen. This knowledge goes beyond exams – use it to understand current events, explore solutions, and contribute to a more sustainable world. Access class 10 geography chapter 1 question answer PDF download and keep learning!

NCERT Solutions for Class 10 Geography- Other Chapter-wise Links for FREE PDF

Dive into our FREE PDF links offering chapter-wise NCERT solutions prepared by Vedantu Experts, to help you understand and master the social concepts.

NCERT Solutions for Social Science Class 10 Geography other Chapter-wise Links

Related Important Links for Class 10 Geography

S. No

Important Links for Class 10 Social Science

1.

 

2.

3.

4.

arrow-right

FAQs on NCERT Solutions for Class 10 Geography Chapter 1 Resources and Development

1. Why do we Need resources and development class 10 PDF?

Class 10 Social Science Resources and Development Solutions are important because they help students understand the chapter in detail and prepare for their final exams. The chapter covers topics such as resource planning in India, identification and inventory of resources, evolving a planning structure, and appropriate technology, skill and institutional set up for implementing resource development plans. The NCERT Solutions for Class 10 Social Science give students a further advantage to answer different questions that may come in their exams from various topics. These solutions provide clear and understandable answers in an engaging and easy-to-learn manner. Students can download these solutions for free from Vedantu website.

2. How can Vedantu Help You understand Geography Chapter 1 class 10?

Vedantu can help you in understanding Class 10 Social Science Contemporary India Ch 1 by providing NCERT Solutions for the chapter. These solutions are crafted by subject matter experts and are available for free download on Vedantu's website. The solutions are provided in a simple language that is easily understandable and helps students understand the chapter in detail. Vedantu also provides chapter-wise NCERT Solutions for Class 10 Social Science, including History, Geography, Political Science and Economics, which can help students prepare for their exams and score good marks. Students can download these solutions in PDF format from Vedantu's website or app and revise the complete syllabus to score more marks.

3. What about geography class 10, chapter 1  from Contemporary India?

Chapter 1 of Class 10 Contemporary India is about Resources and Development. This chapter introduces students to resources and their classification. It covers topics such as resource planning in India, identification and inventory of resources, evolving a planning structure, and appropriate technology, skill, and institutional set up for implementing resource development plans. The chapter also discusses the three stages of resource planning in India, which are identification and inventory of resources, evolving a planned structure, and matching the resource development plans with overall national development plans. Students can find NCERT Solutions and notes for this chapter on our Vedantu website.

4. How can you categorise resources based on ownership from Geography class 10 chapter 1 question answers?

On the basis of ownership, resources can be classified as (1) Individual or Community-owned resources and (2) Natural or International resources. Individual resources are the resources that are privately owned by individuals whereas community-owned resources are those that belong to the entire community and are accessible to all the members. All the resources such as minerals, water resources, forests, etc. found in a nation and up to 12 nautical miles from the coastline belong to the nation. Oceanic resources beyond 200 nm are regulated by international institutions. 

5. What are the obstacles to the development of resources as discussed in class 10 geography chapter 1 question answers?

Resources are vital for our survival, sustenance and development, and well-being. Human beings have used the resources indiscriminately over the course of history. There are various problems that need to be overcome to ensure optimum development of resources. These include depletion of resources, resources accumulation in a few hands, and indiscriminate exploitation of resources. Consequently, the climatic conditions across the world have deteriorated. 

6. Does resource availability ensure a region's development from resources and development class 10?

Resources are crucial for our survival and the development of quality of life but the availability of resources does not automatically translate into the development of the region. There are regions that are rich in resources but are not entirely developed. Availability of resources is important. But what is equally important is the availability of necessary technology and institutions to ensure that resources are used in an optimum, efficient, and judicious way.

7. Where can I find solutions to class 10 Geography Chapter 1?

Vedantu  is one of the websites on which you can trust. The Vedantu website will help you to tackle all your doubts and concerns related to important questions or solutions smartly. The solutions are provided in PDF format and are accessible even on the go. These materials are also available on the Vedantu app. All the material is free of cost. The answers are crafted to satisfy every demand of the question and allow you to find for yourself the best way of presenting the answer and score an extra mark in the final battle. 

8. What are some key topics covered in the NCERT resources and development class 10 questions and answers?

Classification of resources (natural, human-made, renewable, non-renewable)

The importance of resource development for a nation's economic and social progress

The concept of sustainable development and its principles

Challenges of resource depletion and environmental degradation

Strategies for responsible resource management and conservation practices.

NCERT Solutions for Class 10 Geography Chapter 1 Free PDF Download

Ncert solutions for class 10 geography chapter 1 – resources and development.

The subject geography is not as easy as it looks. Also, the students take the subject very lightly. But, the students forget the fact that it is one of the most scoring subjects of Social Science. Our NCERT Solutions for Class 10 Geography Chapter 1 will guide you with the content of the chapter. Also, it will assist students in understanding the topics of the Chapter.

The NCERT Solutions for Class 10 Geography Chapter 1 aims to help students understand the Chapter in an easy way. Also, our expert makes them using their expert knowledge and experience. Download the Toppr app for  Android  and  iOS  or signup for free.

Download  NCERT Solutions for other subjects here.

Download  NCERT Solution for  Class 7 Chapter-wise here.

Download  NCERT Solutions for other chapters of Geography here.

CBSE Class 10 Geography Chapter 1 – Resources and Development NCERT Solutions

First of all, let’s discuss resources. Resources refer to all those things present in the environment to satisfy our need is Resources. Also, the Chapter tells how we use these resources for development.

assignment 1 geography class 10

Sub-topics covered under NCERT Solutions for  Class 10 Geography Chapter 1

1.1 Types of Resources- The topic discusses the various types of resources which mention below:

1.1.1 On the basis of Origin

  • Biotic Resources-like flora and fauna.
  • Abiotic Resources- like rocks and metals.

1.1.2 On the basis of Exhaustibility

  • Renewable Resources- Like solar and wind energy, water, etc.
  • Non-Renewable Resources- Like CNG, petroleum, coal, etc.

1.1.3 On the basis of Ownership

  • Individual Resources- Owned by an individual.
  • Community Owned Resources- Owned by the community such as burial ground, Village ponds, etc.
  • National Resources- The resources within the country.
  • International Resources- Those resources which are outside the Exclusive Economic Zone of the country.

1.1.4 On the basis of Status of Development

  • Potential Resources- The resources which are present in an area but cannot be utilized.
  • Developed Resources- There quantity and quality has been resolute for utilization.
  • Stock- Refers to those resources which cannot be used due to lack of suitable technology.

1.2 Development of Resources-

This topic deals with the way by which we can find a sustainable way of using resources. Also, the ways by which we can stop misuse or overuse of resources.

1.3 Resource Planning- This topic discusses the plan which we can make for using resources. Also, how a country like India (rich in resources) can utilize this plan.

1.4 Resource Planning in India- This topic deals with a complex process that involves identification, evolving and matching of resources. Also, how a country like India can make and execute such a plan successfully.

1.4.1 Conservation of Resources- Discusses the ways by which irrational and over-utilization of resources can be checked.

1.5 Land Resources- Talks about the land which is the most useful and important resource. Also, the ways to use this finite resource are carefully planned.

1.6 Land Utilization- ways in which we can use forest, cultivation, fallow Lands, etc.

1.7 Land Use Pattern in India- Discuss the various land use pattern like agriculture, forest, barren, non-agricultural, etc.

1.8 Land Degradation and Conservation- Discusses the ways of ruining of land. On the other hand, discuss the ways to preserve the land.

1.9 Soil as a Resource- This topic deals with soil which is the key natural renewable resource. Above all, it supports all human life on earth. Furthermore, the topic covers various factors which affect the soil.

1.10 Classification of Soil-

This topic further divides into several subtopics which describe the various types of soils mentioned below:

  • Alluvial Soil
  • Reds and yellow Soil
  • Laterite Soil
  • Forest Soils

1.11 Soil Erosion and Soil Conservation- This topic discusses the ways by which the topmost layer get degraded. And in the second part discuss the ways of conserving this soil.

You can download NCERT Solutions for Class 10 Geography Chapter 1 by clicking on the download button below

assignment 1 geography class 10

Download Toppr – Best Learning App for Class 5 to 12

Toppr provides free study materials, 1000+ hours of free video lectures, live doubts solving, and more. For online assistance and live doubt clearance, mock test, adaptive learning, simple and easy solutions download the Toppr app for  Android  and  iOS app or signup for free.

Solved Questions For You:

Question 1.  The means for the movement of goods and services from their supply locations to demand locations can be termed as which of the following?

Question 2: Which of the following networks of pipeline bring mineral oil to the refinery of Barauni and petrochemical complex of Haldia?

Question 3: Which of the following mode of transport is fuel-efficient and environment-friendly?

Question 4:  Which of the following is a means of mass communication?

Customize your course in 30 seconds

Which class are you in.

tutor

NCERT Solutions for Class 10

  • NCERT solutions for Class 10 Geography Chapter 3 Free PDF Download
  • NCERT Solutions for Class 10 Science Chapter 2 Free PDF Download
  • NCERT Solutions for Class 10 Mathematics Free PDF Download
  • NCERT Solutions for Class 10 History Chapter 6 Free PDF Download
  • NCERT Solutions for Class 10 History Chapter 1 Free PDF Download
  • NCERT Solutions for Class 10 History Chapter 2 Free PDF Download
  • NCERT Solutions for Class 10 History Chapter 4 Free PDF Download
  • NCERT Solutions for Class 10 Social Science Chapter 5 Free PDF Download
  • NCERT Solutions for Class 10 Social Science Chapter 7 Free PDF Download
  • NCERT Solutions for Class 10 Social Science Chapter 8 Free PDF Download

Leave a Reply Cancel reply

Your email address will not be published. Required fields are marked *

Download the App

Google Play

Physics Wallah

NCERT Solutions for Class 10 Social Science Geography Chapter 1 Resources and Development

Here, we have provided NCERT Solutions for Class 10 Social Science Geography Chapter 1. Students can view these NCERT Solutions for Class 10 Social Science Geography Chapter 1 before exams for better understanding of the chapter.

Photo of author

April 7, 2024

NCERT Solutions for Class 10 Social Science Geography Chapter 1

Table of Contents

NCERT Solutions for Class 10 Social Science Geography Chapter 1:  Students can use NCERT Solutions Class 10 Geography Chapter 1 Resources and Development as a valuable study tool to help them get ready for the CBSE exams.

Students will learn the answer writing style by reading through these solutions, which will help them receive higher exam scores. By including all the pertinent information, they will also comprehend how to write their responses.

Checkout Class 10th Combos from PW Store

Experts in the field have prepared these NCERT Class 10 Solutions, which aid students in properly preparing for their Social Science exams.

Checkout Class 10th Books from PW Store

NCERT Solutions for Class 10 Social Science Geography Chapter 1 Overview

NCERT Solutions for Resource and Development Class 10 Chapter 1 of Social Science Geography. “Resource”: In this chapter students will learn thoroughly about resources.

Checkout Class 10th Modules from PW Store

Anything in our surroundings that we can use to meet our requirements is referred to as a resource. It must be economically viable, culturally acceptable, and technologically accessible.

NCERT Solutions for Class 10 Social Science Geography Chapter 1 PDF

Our website provides a convenient way to download the NCERT Solution for Class 10 Social Science  Contemporary India Resources and Development in PDF format.Any student may readily understand the straightforward language used in the NCERT Solution.

Checkout Class 10th Sample Papers from PW Store

You will therefore become more confident in your writing abilities and regularize your study according to the requirements of the exam by consulting this answer book. Get the expertly chosen NCERT Solutions for Class 10 Social Science Contemporary India from here. Science students will also find the solutions chosen by our master teachers to be quite helpful if they are looking for Class 10 Science NCERT Solutions.

Checkout Class 10th Previous Year Papers from PW Store

NCERT Solutions for Class 10 Social Science Geography Chapter 1

1. Multiple choice questions.

(i) Which one of the following types of resource is iron ore?

(a) Renewable

(d) Non-renewable

Checkout Class 10th Question Bank from PW Store

Non-renewable

(ii) Under which of the following type of resources can tidal energy not be put?

(a) Replenishable

(b) Human-made

(c) Abiotic

(d) Non-recyclable

Replenishable

(iii) Which one of the following is the main cause of land degradation in Punjab?

(a) Intensive cultivation

(b) Deforestation

(c) Over-irrigation

(d) Overgrazing

Over-irrigation

(iv) In which one of the following states is terrace cultivation practised?

(b) Plains of Uttar Pradesh

(c) Haryana

(d) Uttarakhand

Uttarakhand

(v) In which of the following states is black soil predominantly found?

(a) Jammu and Kashmir

(b) Maharashtra

(c) Rajasthan

(d) Jharkhand

  • Maharashtra

2. Answer the following questions in about 30 words .

(i) Name three states having black soil and the crop which is mainly grown in it.

3 states are

  • Madhya Pradesh

The crop grown is cotton.

(ii) What type of soil is found in the river deltas of the eastern coast? Give three main features of this type of soil.

Alluvial soil is the kind of soil that is found in river deltas.

Because of its high fertility, it is suitable for crop cultivation. It is made up of different amounts of clay, silt, and sand. Alluvial soil is rich in phosphoric acid, lime, and potash, all of which are beneficial to the growth of sugarcane and paddy.

(iii) What steps can be taken to control soil erosion in hilly areas?

The primary methods that are applicable are listed below.

Ploughs with contours cultivating terraces Grass is permitted to grow in strips between the crops. Strip cropping is the term for this technique.

(iv) What are the biotic and abiotic resources? Give some examples.

Biotic resource

These are materials derived from the biosphere. These materials are living Plants, animals, fish, people, cattle, etc. are a few examples.

An abiotic source

These resources are made up of dead things. Water, minerals, metals, wind, solar energy, etc. are a few examples.

3. Answer the following questions in about 120 words.

(i) Explain the land use pattern in India and why has the land under forest not increased much since 1960-61.

Answer:  The physical characteristics of topography, climate, and soil types, as well as the human elements of population density, technological prowess, culture, and customs, all influence how land is used.

States differ significantly in their net sown area patterns. In Punjab and Haryana, it makes up more than 80% of the entire area, while in Arunachal Pradesh, Mizoram, Manipur, and the Andaman Nicobar Islands, it makes up less than 10%.

As stated in the National Forest Policy (1952), the country’s forest area is much less than the targeted 33% of its total land area. It was thought to be crucial for preserving the ecological equilibrium. Wasteland is a term used to describe a portion of the land used for purposes other than agriculture, such as roads, railroads, industries, and communities.

It contains desert, rocky, and arid regions. Land degradation is the result of long-term, continuous exploitation of a resource without the necessary conservation and management actions.

(ii) How has technical and economic development led to more consumption of resources?

This is due to a number of factors.

Extensive manufacturing resulted in excessive resource usage. The development of technology increased resource exploitation. Increased access to health and medical resources resulted in massive resource use.

Benefits of NCERT Solutions for Class 10 Social Science Geography Chapter 1

One of the fundamental subjects that demands for a well-reasoned interpretation together with accurate comprehension and analysis to test your knowledge is social science.

Class 10 students are extremely competitive and under constant pressure to perform well on their final examinations as the results will determine their course of study moving forward.

In addition to textbooks, students also need solution books, which will save them time while also assisting them in answering the questions.

The seven extremely large subtopics in Class 10 Social Science Contemporary India Ch 1 NCERT Solutions require careful attention to acquire, comprehend, and retain for exams.

NCERT Solutions for Class 10 Social Science Geography Chapter 1 FAQs

Class 10 Geography Chapter 1 - Resources and Development introduces students to the concept of resources and their classification.

To get good marks in Geography, students must devote sufficient time and attention to each chapter.

Yes, Geography class 10 is easy.

card-img

NCERT Solutions for Class 10 Social Science History Chapter 8 Novels Society and History

NCERT Solutions for Class 10 Social Science Geography Chapter 2 Forest and Wildlife Resources

right adv

.st1{display:none} Related Articles

  • CBSE Class 12th Sample Papers 2024-25 OUT Now Download PDF
  • CBSE Class 12 Chemistry Notes Chapter 13 Amines
  • CBSE Class 12 Physics Notes Check Chapter Wise Notes PDF
  • UP Board Class 12 Syllabus 2024-25 Subject Wise PDF Download
  • CBSE Board Exam 2025 10 and 12 Registration Opens, steps to Register
  • CBSE Class 12 Physics Notes Chapter 14 Semiconductor Devices & Communication System
  • CBSE Class 12 Physics Notes Chapter 13 Nuclei PDF Download
  • CBSE Class 12 Physics Notes Chapter 12 Atoms
  • CBSE Class 12 Physics Notes Chapter 11 Dual Nature of Matter and Radiation
  • CBSE Class 12 Chemistry Notes Chapter 12 Aldehydes, Ketones and Carboxylic Acids

bottom banner

Have doubts?

Our support team will be happy to assist you!

call icon

  • School Guide
  • Class 10 Syllabus
  • Maths Notes Class 10
  • Science Notes Class 10
  • History Notes Class 10
  • Geography Notes Class 10
  • Political Science Notes Class 10
  • NCERT Soln. Class 10 Maths
  • RD Sharma Soln. Class 10
  • Math Formulas Class 10

CBSE Notes Class 10 Geography Chapter 1- Resources and Development

Resources and Development is the first chapter in CBSE Class 10 Geography , which discusses the concept of resources and their various forms. The chapter covers the different types of resources such as natural, human-made, and human resources, and their utilization for the development of society. It also discusses the process of resource development and planning in India, focusing on the need for sustainable development. 

Additionally, the chapter covers the topics of soil conservation and erosion, highlighting the causes and effects of soil erosion and various methods of soil conservation. Overall, this chapter provides an overview of the various resources available on Earth and their importance in the development of human society. The Resources and Development notes cover all the necessary topics that are extremely important for the board examination 2024. It can save you time and will positively affect your result.

Class 10 Geography Chapter 1: Resources and Development

Resources and their Types
Development of Resources
Resource Planning
Land Resources, Land Utilization and Land Use Pattern in India
Land Degradation and Conservation
Soil as a Resource and Classification of Solids
Soil Erosion and Soil Conservation
1. Resources 

Everything in the environment around us that can be used to fulfil our requirements, and is technologically accessible, economically practical, and culturally acceptable, is referred to as a ‘Resource’. Human actions also function as resources. Humans are necessary components of resources. They convert the components in our surroundings into resources and consume them.

Classification of Resources

Resources can be classified in the following ways and types:

  • On the basis of origin – biotic and abiotic
  • On the basis of exhaustibility – renewable and non-renewable
  • On the basis of ownership – individual, community, national and international
  • On the basis of the status of development – potential, developed stock, and reserves.

Classification of Resources

1. On the Basis of Origin- Biotic and Abiotic

Biotic Resouces: These are taken from the biosphere and contain life, such as humans, plants and wildlife, fisheries, ruminants, and so on. 

Abiotic Resouces: The abiotic resource is anything that is made up of nonliving items such as rocks and metals.

2. On the Basis of Exhaustibility- Renewable and Non-Renewable

Renewable Resources: Renewable or replenishable resources are those that can be replaced or reproduced by physical, chemical, or mechanical processes, such as solar and wind energy, water, forests and wildlife, and so on. Renewable resources can be further divided into continuous or flow.

Non-renewable Resources: These occur throughout a wide range of geological periods. These resources take millions of years to form. Minerals and fossil fuels are examples of resources that take millions of years to create. Some resources take like metals are recyclable, whereas some fossil fuels cannot be recycled and get exhausted.

3. On the Basis of Ownership- Individual, Community, National, and International

Individual Resources: These are privately held by people. Personal resource ownership includes plantations, pasture fields, ponds, water in wells, and various other things.

Community Resources: These are communal resources that are available to all members of the community, such as grazing fields, burial sites, village ponds, public spaces, picnic areas, playgrounds, and so on. 

National Resources: Technically, the country owns all of the resources. The nation has the legal authority to acquire private property for the public interest. The nation owns all minerals, water resources, forests, animals, and land within its political limits, as well as the oceanic region up to 12 nautical miles (22.2 km) from the shore, referred to as territorial water and its resources.

International Resources: Some resources are regulated by international institutions. Beyond 200 nautical miles of the Exclusive Economic Zone, marine resources belong to open waters, and no one country can use them without the approval of an international authority.

4. On the basis of the Status of Development- Potential, Developed Stock, and Reserves

Potential Resources: These resources exist in a region but have not been used. For example, the western areas of India, notably Rajasthan, and Gujarat offer huge potential for wind and solar energy development, but they have yet to be completely developed.

Developed Resources: These are resources that have been surveyed and their quality and quantity determined for use. The development of resources is determined by technologies and their level of sustainability.

Stock: Stock refers to materials in the environment that have the potential to meet human requirements but lack the required technology to accomplish so. For example, Hydrogen is a valuable source of energy. However, we need advanced technology to put it to use.

Reserves: Reserves are a subset of the stock that can be used with the technology ‘know-how’ but has not yet been used. These can be utilized to satisfy requirements in the future. For example, Water in dams, woods, and other sources of water is a reserve that can be utilized in the future.

Read More: Resources and its Types
2. Development of Resources

Resources are essential for human life; however, they have been utilized indiscriminately by humans, resulting in problems such as:

  • The centralized control of resources in a few hands split society into two groups, rich and poor.
  • Consumption of resources to satisfy the avarice of a few people.
  • It has resulted in worldwide environmental issues such as global warming, ozone depletion, pollution, and land degradation.

A fair utilization of resources has become critical for long-term quality of life and world peace. However, its abolition by a few individuals or countries may endanger the planet. Resource planning is important for the sustainable existence of different forms of life and Sustainable Economic Development refers to the development which takes place without damaging the environment, and development happening at the present shouldn’t compromise with the generations to come.

3. Resource Planning

The commonly acknowledged technique for prudent resource utilization is planning. There are locations in India that are rich in certain sorts of resources but poor in others.  There are certain locations that can be called self-sufficient in terms of resource availability, while others have severe shortages of critical resources. This necessitates resource planning that is balanced at the national, state, regional, and local levels.

Resource Planning in India

Resource planning is a complicated process that includes:

  • Identifying and inventorying resources across the country. This includes collecting data, mapping, and both quantitative and qualitative resource estimates and measurements.
  • Developing a planning framework that is equipped with the necessary technology, skills, and institutional infrastructure to carry out resource development plans.
  • Linking resource development strategies with long-term national development goals.

From the First Five Year Plan, released after independence, India has made coordinated efforts to achieve resource planning goals. The availability of resources alone may hinder progress in the absence of equivalent advances in technology and institutions. Resources when accompanied by proper technical progress and institutional adjustments, contribute to development.

Resource conservation at many levels is critical for overcoming irrational consumption and over-utilization of resources.

4. Land Resources

Land is a vital natural resource, but it is a finite property; thus, it is critical to use the existing land for varied purposes with careful consideration. India contains land with a range of relief features, including mountains, plateaus, plains, and islands.

Land Resources

Land Resources

Land Utilization

The following uses are made of land resources:.

  • Barren and Wasteland
  •  Land put out for non-agricultural uses
  • Fallow lands
  • Other uncultivated lands which exclude fallow land
  • Net Sown area

Land Use Pattern in India

The usage of land is influenced by both physical and human causes:

  • Physical Factors: These refer to the factors such as topography, climate, and soil types.
  • Human Factors: The human factors include population density, technological capability, culture, traditions, etc.

General Land Use Categories

General Land Use Categories

Wasteland refers to the land which has been put out for other non-agricultural uses and includes rocky, arid, and desert regions, roads, railways, and so forth. The continuous use of the land resource over a long period of time, without taking appropriate measures for management and conservation, results in what is known as land degradation.

Read More: Land Utilization and Land Use Pattern in India

Land Degradation and Conservation Measures

Human activities including deforestation, overgrazing, mining, and extraction have not only caused land degradation but have also accelerated the rate at which natural processes cause land harm.

There are several approaches for resolving land degradation issues:

  • Afforestation and adequate grazing maintenance.
  • Planting of plant shelter zones.
  • Sand dunes are stabilized by the growth of prickly shrubs.
  • Wastelands must be managed properly.
  • Mining activity supervision.
  • After-treatment release and removal of industrial wastewater and wastes.
Read More: Land Degradation and Conservation
5. Soil as a Resource

The most significant renewable natural resource is soil. Various natural factors such as temperature variations, the acts of running water, wind, glaciers, decomposer activities, and chemical and organic changes that occur in the soil all contribute to the development of soil:

  • Soil takes millions of years to be formed up to only a few cm in depth. There are various factors and forces of nature such as temperature, actions of running water, wind, etc. which contribute to the formation of soil.
  • The parent rock or bedrock, climate, vegetation, etc. are forms of life and time, important in the formation of soil.
  • Chemical and also organic changes also play important roles in soil formation.
  • Soil consists of both organic as well as inorganic materials.
6. Classification of Soils

India has a wide range of environmental characteristics, landforms, climate zones, and flora kinds. These have aided in the formation of distinct soil formations, color, thickness, texture, age, chemical as well as physical properties, soils of India can be classified as:

Alluvial Soils

  • This is the most widely distributed and significant soil. Alluvial soil covers the whole northern plains. 
  • It can also be found in Rajasthan, Gujarat, and the eastern coastal plains, namely in the deltas of the Mahanadi, Godavari, Krishna, and Kaveri rivers.
  • These were deposited by three major Himalayan river systems: the Indus, Ganga, and Brahmaputra.
  • Sand, silt, and clay are the components of alluvial soil.
  • Based on age, Alluvial soils can be classified as:
  • Old Alluvial (Bangar): The Bangar soil has a higher concentration of Kanker nodules than that of Khadar.
  • New Alluvial (Khadar): It has more fine particles and is more fertile as compared to Bangar.
  • Alluvial soils are mostly very fertile and contain an adequate amount of potash, phosphoric acid, and lime, ideal for the growth of sugarcane, paddy, wheat, and other pulses crops.

Alluvial Soil

Alluvial Soil

  • These black soils are also referred to as regur soils. These are great for cotton cultivation.
  • The parent rock material, as well as the climatic conditions, are essential elements in the production of black soil.
  • The soil extends southeast along the Godavari and Krishna basins and covers the plateaus of Maharashtra, Saurashtra, Malwa, Madhya Pradesh, and Chhattisgarh.
  • The black soils are composed of exceedingly fine clayey minerals. They are widely known for their ability to retain moisture.
  • They are high in soil nutrients such as calcium carbonate, magnesium, potash, and lime.
  • This type of soil is found mostly in the Deccan Trap region and spread over the northwest Deccan plateau and is also made up of lava flows.
  • The soil has a sticky nature and is difficult to work with unless it is tilled immediately after the first shower during pre-monsoon periods.

Black soil

Red and Yellow Soils

  • Red soil forms on crystalline igneous rocks in low-rainfall sections of the Deccan plateau’s eastern and southern regions.
  • Yellow and red soils may be found in regions of Odisha, Chhattisgarh, the southern Ganga plain, and the Piedmont zone of the Western Ghats.
  • The reddish color of these soils is caused by iron diffusion in crystalline and metamorphic rocks.
  • When it is hydrated, it has a yellow appearance.

Red and Yellow Soil

Red and Yellow Soil

Laterite Soils

  • Laterite soil forms in tropical and subtropical climates with alternating wet and dry seasons. 
  • This soil is the product of heavy rain’s extensive leaching.
  • They are mainly deep to variable depths, acidic in nature (pH 6.0), and often low in plant nutrients. This type of soil is generally found in the southern states, the western Ghats area of Maharashtra, Odissa, some portions of West Bengal, and the Northeastern states.
  • The soil supports deciduous and evergreen woods, but it is deficient in humus.
  • This soil is extremely beneficial for growing tea and coffee.

Laterite Soil

Laterite Soil

  • Arid soils have a sandy texture and are salty in character. 
  • Evaporation occurs faster due to the dry environment and high temperatures, and the soil lacks humus and moisture.
  • Kankar occupies the bottom layer of the soil because of the rising calcium concentration downwards.
  • The colour of arid soils ranges from red to brown.

Arid Soils

Forest Soils

  • These soils are found in hilly and mountainous areas where there is enough rainforest.
  • The valley sides are loamy and silty, whereas the top slopes are coarse-grained.
  • The texture of the soil changes depending on the alpine environment in which it is generated.
  • Fertile soils may be found in the lower valleys, particularly on river terraces and alluvial fans.

Forests soils

Forests Soils

The map below shows the distribution of soil in India:

Major types of soil

Major types of soil

Read More: Soil Formation
7. Soil Erosion and Soil Conservation

Soil erosion is defined as the degradation of the soil cover and subsequent washing away. Soil erosion is caused by human activities such as deforestation, overgrazing, building, and mining, as well as natural processes such as wind, glaciers, and water. Running water breaks through the clayey soils, forming deep channels known as gullies.  

The land has become unsuited for agriculture and is referred to as bad land. Such regions are known as ravines in the Chambal basin. Water can flow in a sheet across huge portions of the slope at times. This is referred to as sheet erosion. Wind erosion occurs when the wind sweeps loose dirt off the flat or sloping ground. Defective agricultural techniques also contribute to soil erosion.

Read More: Soil Erosion

Different Ways for Soil Conservation

  • Ploughing following contour lines slows the flow of water down the hills. This is known as contour ploughing.
  • Terrace cultivation helps to prevent erosion. This form of farming is practised in the Western and Central Himalayas.
  • When a vast field is split into strips and grass is allowed to grow in between the crops. This then splits up the wind’s power. Strip cropping is the name given to this technique.
  • Planting lines of trees to provide cover aids in the stability of sand dunes in the western Indian desert. Shelter Belts are rows of such trees.

FAQs on CBSE Class 10 Geography Chapter 1: Resources and Development

Q 1. what is a resource.

Everything in the environment around us that can be used to fulfill our requirements, and is technologically accessible, economically practical, and culturally acceptable, is referred to as a ‘Resource’.

Q 2. Name the types of Resources.

The types of resources include Natural, Human and Human Made resources.

Q 3. Why are resources important?

Resources are important as without resources none of our needs are satisfied and all the raw materials are obtained mostly from resources.

Please Login to comment...

Similar reads.

  • School Geography
  • School Learning
  • Social Science
  • Best Twitch Extensions for 2024: Top Tools for Viewers and Streamers
  • Discord Emojis List 2024: Copy and Paste
  • Best Adblockers for Twitch TV: Enjoy Ad-Free Streaming in 2024
  • PS4 vs. PS5: Which PlayStation Should You Buy in 2024?
  • 10 Best Free VPN Services in 2024

Improve your Coding Skills with Practice

 alt=

What kind of Experience do you want to share?

Study Rankers

NCERT Solutions for Class 10th: Ch 1 Resources and Development Geography

Ncert solutions for class 10th: ch 1 resources and development geography social studies (s.st), contact form.

assignment 1 geography class 10

LIVE Course for free

assignment 1 geography class 10

  • Ask a Question

Join Bloom Tuition

NCERT Solutions Class 10 Social Science Geography Chapter 1 Resources and Development.

assignment 1 geography class 10

NCERT Solutions Class 10, Social Science, Geography, Chapter 1, Resources and Development.

To learn the fundamentals of this chapter, as well as to prepare for CBSE exams and competitive exams refer to these NCERT Solutions. These solutions have been created by subject matter experts to provide an in-depth analysis of all the concepts covered in this chapter. This study material is based on the most recent CBSE syllabus.

In these  NCERT Solutions for Class 10   Social Science , we have discussed all types of  NCERT intext questions, exercise questions as well as multiple choice type questions.

Concepts covered in Class 10 Social Science - Geography chapter 1 Resources and Development are :

  • Resources and Development are Changing Land-use Pattern
  • Land Degradation and Conservation Measures
  • Types - Natural and Human
  • Need for Resource Planning
  • Concept of Land as a Resource
  • Types of Soil, Natural Resources
  • Soil Erosion
  • Soil Conservation
  • Changing Land-use Pattern
  • Types of Soil
  • Natural Resources
  • Soil Conservation.

Our NCERT Solutions for  Class 10 Social Science   provide detailed explanations to assist students with their homework and assignments. Proper command and ample practice of topic-related questions provided by our NCERT solutions is the most effective way to achieve full marks in your exams. Begin studying right away to ace your exams.

Now all the solutions and practice questions are at your fingertip to get started.

  • ncert solutions
  • ncert solutions class-10
  • ncert solutions class-10 social science
  • resources and development

Please log in or register to add a comment.

assignment 1 geography class 10

1 . Multiple choice questions.

(i) Which one of the following is the main cause of land degradation in Punjab? (a) Intensive cultivation  (b) Deforestation (c) Over irrigation (d) Overgrazing

The correct option is :  (c) Over irrigation

(ii) In which one of the following states is terrace cultivation practised? (a) Punjab (b) Plains of Uttar Pradesh (c) Haryana (d) Uttarakhand

The correct option is :  (d) Uttarakhand

(iii) In which of the following states black soil is predominantly found? (a) Uttar Pradesh (b) Maharashtra (c) Rajasthan (d) Jharkhand

The correct option is :  (b) Maharashtra

(iv) Which one of the following type of resource is iron ore? (a) Renewable (b) Biotic (c) Flow (d) Non-renewable

The correct option is :  (d) Non-renewable

(v) Under which of the following type of resource can tidal energy be put? (a) Replenishable (b) Human-made (c) Abiotic (d) Non-recyclable

The correct option is :  (a) Replenishable

2 . Answer the following questions in about 30 words.

(i) Name three states having black soil and the crop which is mainly grown in it.

Solution: 

Maharashtra, Madhya Pradesh and Chhattisgarh Black soil is ideal for growing cotton.

(ii) What type of soil is found in the river deltas of the eastern coast? Give three main features of this type of soil.

Alluvial soil Alluvial soils are very fertile. They are ideal for growing sugarcane, wheat and paddy. As it has more kankar nodules, old alluvial is less fertile than new alluvial.

(iii) What steps can be taken to control soil erosion in the hilly areas?

In hilly areas, soil erosion can be controlled by ploughing across contour-lines, making use of terrace farming techniques and using strips of grasses to check soil erosion by wind and water.

(iv) What are the biotic and abiotic resources? Give some examples.

The resources which are obtained from the biosphere and have life are called Biotic Resources. For example, animals and plants including human beings. Abiotic Resources: The resources which are composed of non-living things are called Abiotic Resources. For example, water, minerals, metals, wind, solar energy etc.

3. Answer the following questions in about 120 words.

(i) Explain land use pattern in India and why has the land under forest not increased much since 1960-61?

Land resources in India are primarily divided into agricultural land, forest land, land meant for pasture and grazing, and wasteland. Waste land includes rocky, arid, desert areas and land used for other non-agricultural purposes such as housing, roads, and industry. According to recent data, about 54% of the total land area is cultivable or fallow, 22.5% is covered by forests, and 3.45% is used for grazing. The rest is a wasteland, with traces of miscellaneous cultivation.

The improper use of forest land has degraded the available land area and has made the conservation of forests difficult. Human activities such as deforestation, mining and quarrying have contributed to the slow growth rate of forests. Thus, land under forest has increased by only about 4% since 1960-61.

(ii) How have technical and economic development led to more consumption of resources?

Technical and economic development has led to more consumption of resources on account of various factors. In colonial times, imperial powers would use their technological and economic superiority to establish control over other countries and thereby gain access to the latter’s resources. Now, one country’s resources became accessible to the citizens of its colonial ruler too, leading to increased consumption. Also, because of technical and economic progress, populations are increasing due to low mortality at all ages. With the development of medicine and health care, fewer people die due to accidents, diseases, childbirth etc. This, too, has contributed to higher consumption of resources.

Multiple Choice Questions

1. Resources which are surveyed and their quantity and quality have been determined for utilisation is known as (a) Potential resources (b) Stock (c) Developed resources (d) Reserves

The correct option is :  (c) Developed resources

2. Which one of the following soil is ideal for growing cotton? (a) Regur soil (b) Laterite soil (c) Desert soil (d) Mountainous soil

The correct option is :  (a) Regur soil

3. In which of the following states is overgrazing responsible for land degradation? (a) Jharkhand and Orissa (b) Madhya Pradesh and Rajasthan (c) Punjab and Haryana (d) Kerala and Tamil Nadu

The correct option is :  (b) Madhya Pradesh and Rajasthan

4. Which one of the following statements is true about the term resources? (a) Resources are free gifts of nature. (b) They are the functions of human activities. (c) All those things which are found in nature. (d) Things which cannot be used to fulfill our needs.

The correct option is :  (b) They are the functions of human activities.

5. Which one of the following types of the resource is iron ore? (a) Renewable (b) Biotic (c) Flow (d) Non-renewable

6. Under which of the following types of resource the tidal energy can be put? (a) Replenishable (b) Human-made (c) Abiotic (d) Non-recyclable

7. Soil formed by intense leaching is (a) Alluvial soil (b) Red soil (c) Laterite soil (d) Desert

The correct option is :  (c) Laterite soil

8. Fallow land refers to (a) land not under cultivation. (b) land with many gullies. (c) a fertile land. (d) cultivable land not cultivated for a season to regain its fertility.

The correct option is :  (d) cultivable land not cultivated for a season to regain its fertility.

9. Method of growing long strips of grass between the crops refers to (a) Contour ploughing (b) Terrace farming (c) Strip cropping (d) Crop rotation

The correct option is :  (c) Strip cropping

10. Resources which are found in a region, but have not been utilised. (a) Renewable (b) Developed (c) National (d) Potential

The correct option is :  (d) Potential

11. Which of the following factors involves the transformation of things into a resource ? (i) Physical environment (ii) Technology (iii) Human beings (iv) Institutions (a) (i) and (ii) (b) (ii) and (iii) (c) (i) and (iv) (d) All of above

The correct option is :  (d) All of above

12. Renewable resources are those (a) which cannot be renewed (b) which are accessible (c) which are developed (d) which are renewed by physical, chemical or mechanical processes.

The correct option is :  (d) which are renewed by physical, chemical or mechanical processes.

13. Which one of the following is not a community resource ? (a) Public parks (b) A library (c) A car (d) A community hall

The correct option is :  (c) A car

14. Territorial waters of India extends to (a) 10 Nautical miles (b) 15 Nautical miles (c) 12 Nautical miles (d) 1900 kilometres

The correct option is :  (c) 12 Nautical miles

15. Find out which one of the following is a stock? (a) Biofuels (b) Coal (c) Solar energy (d) Hydro-electricity

The correct option is :  (a) Biofuels

16. The first International Earth Summit was held in (a) Geneva (b) New York (c) Japan (d) Rio de Janeiro

The correct option is :  (d) Rio de Janeiro 

17. “There is enough for everybody’s need but not for anybody’s greed”. Who said this ? (a) Jawahar Lai Nehru (b) Atal Bihari Vajpai (c) M. K. Gandhi (d) Sunder Lai Bhauguna

The correct option is :  (c) M. K. Gandhi

18. The area brought under cultivation in a year is called ……………. (a) Fallow land (b) Cultivable (c) Net sown area (d) Gross sown area

The correct option is :  (c) Net sown area

19. I am the most widespread soil, covering the Northern Plains and Eastern Coastal Plains-who am I ? (a) Black soil (b) Forest soil (c) Red soil (d) Alluvial soil

The correct option is :  (d) Alluvial soil 

20. Resources which are non-renewable but can be recycled are called (a) Renewable resources (b) Non-renewable resources (c) Recyclable resources (d) Biotic resources

The correct option is :  (c) Recyclable resources

21. The most widespread relief feature of India is (a) Mountains (b) Forests (c) Plains (d) Plateaus

The correct option is :  (c) Plains

22. The current net sown area of India in 2002-03 is (a) 45 percent (b) 43.4 percent (c) 50 percent (d) 48 percent

The correct option is :  (b) 43.4 percent

23. The state having maximum net sown area in India is …………… (a) Jammu and Kashmir (b) Uttar Pradesh (c) Tamil Nadu (d) Punjab

The correct option is :  (d) Punjab

24. Land left without cultivation for one or less than one agricultural year is called (a) Culturable waste land (b) Current fallow land (c) Waste land (d) None of the above

The correct option is :  (b) Current fallow land

25. The present per cent of area under forests is  (a) 18 percent (b) 22.57 percent (c) 19 percent (d) 11 percent

The correct option is :  (b) 22.57 percent

26. The factor responsible for maximum land degradation is ……………… (a) Human activities (b) Wind (c) Salinity (d) Soil erosion

The correct option is :  (a) Human activities

27. Which agent is responsible for maximum land degradation ? (a) Wind (b) Water (c) Glaciers (d) Overgrazing

The correct option is :  (b) Water

28. Soil is formed by the process of (a) Denudation (b) Gradation (c) Weathering (d) Erosion

The correct option is :  (c) Weathering

29. Supply a technical term for the dead and decomposed material found on the top soil. (a) Bed rock (b) Fossils (c) Humidity (d) Humus

The correct option is :  (d) Humus

30. The old alluvial soil is known as ……………… (a) Bangar (b) Bhabbar (c) Khadar (d) Regur

The correct option is :  (a) Bangar

31. Which of the following statement(s) is true for black soil ? (i) It has larger proportion of clay. (ii) It can retain moisture for a long time. (iii) It develops cracks during summer which helps in aeration. (iv) Cotton grows best in this soil. (a) (i) and (ii) (b) (iii) and (iv) (c) (i) and (iv) (d) All of the above

The correct option is :  (d) All of the above

32. Red colour of soil is due to (a) it is rich in humus. (b) it is rich in iron compounds. (c) it is derived from volcanic origin. (d) it is rich in potash.

The correct option is :  (b) it is rich in iron compounds.

33. The denudation of the soil cover and washing down of soil by various agents are known as ……………… (a) Weathering (b) Gradation (c) Soil erosion (d) Soil conservation

The correct option is :  (c) Soil erosion

34. The land consisting of many gullies and ravines are called ………………. (a) Gully erosion (b) Bed rock (c) V shaped valleys (d) Bad land

The correct option is :  (d) Bad land

35. Terrace cultivation can be used to control soil erosion in (a) Desert regions (b) Hill slopes (c) Valleys (d) Plains

The correct option is :  (b) Hill slopes

36.Strip cropping refers to (a) growing of crops in long strips. (b) growing of trees in long rows. (c) growing of strips of grass in between the crops. (d) ploughing along the contour lines.

The correct option is :  (c) growing of strips of grass in between the crops.

37. Erosion of the top soil when water flows as a sheet over large areas down the slope is called (a) Gully erosion (b) Badlands (c) Soil erosion (d) Sheet erosion

The correct option is :  (d) Sheet erosion

38. Which one of the following statements is correct as regards to international resources ? (a) Resources which are regulated by international institutions. (b) Resources which lie beyond the territorial waters. (c) Resources which are found along the international frontier. (d) Resources which are not yet developed.

The correct option is :  (a) Resources which are regulated by international institutions.

39. Which one of the following methods is ideal for controlling land degradation in coastal areas and in deserts ? (a) Strip cropping (b) Contour ploughing (c) Planting of shelter belts (d) Plugging of gullies

The correct option is :  (c) Planting of shelter belts

40. Which type of soil is suitable for the growth of cashew nut ? (a) Alluvial soil (b) Black soil (c) Red soil (d) Red laterite soil

The correct option is :  (d) Red laterite soil

41. Arid soils are less fertile as (i) it lacks humus and moisture (ii) it has high salt content (iii) it is sandy in nature (iv) it is rich in Iron (a) (i) and (ii) (b) (ii) and (iii) (c) (iii) and (iv) (d) (i) and (iii)

The correct option is :  (a) (i) and (ii)

42. Ploughing along the contour lines can (a) accelerate the flow of water. (b) decelerate the flow of water. (c) accelerate the force of winds. (d) decelerate the force of winds.

The correct option is :  (b) decelerate the flow of water.

43. Bad lands or ravines are found in (a) Chenab basin (b) Chambal basin (c) Ganga basin (d) Godavari basin

The correct option is :  (b) Chambal basin

Find MCQs & Mock Test

  • JEE Main 2025 Test Series
  • NEET Test Series
  • Class 12 Chapterwise MCQ Test
  • Class 11 Chapterwise Practice Test
  • Class 10 Chapterwise MCQ Test
  • Class 9 Chapterwise MCQ Test
  • Class 8 Chapterwise MCQ Test
  • Class 7 Chapterwise MCQ Test

Related questions

assignment 1 geography class 10

  • minerals and energy resources
  • water resources
  • forest and wildlife resources
  • lifelines of national economy
  • manufacturing industries

Welcome to Sarthaks eConnect: A unique platform where students can interact with teachers/experts/students to get solutions to their queries. Students (upto class 10+2) preparing for All Government Exams, CBSE Board Exam , ICSE Board Exam , State Board Exam, JEE (Mains+Advance) and NEET can ask questions from any subject and get quick answers by subject teachers/ experts/mentors/students.

  • All categories
  • JEE (36.8k)
  • NEET (9.4k)
  • Science (789k)
  • Mathematics (256k)
  • Statistics (3.0k)
  • Environmental Science (5.4k)
  • Biotechnology (704)
  • Geography (28.1k)
  • History (28.0k)
  • Civics (11.5k)
  • Economics (19.0k)
  • Disaster Management (65)
  • Commerce (75.0k)
  • Electronics (3.9k)
  • Computer (22.0k)
  • Artificial Intelligence (AI) (3.3k)
  • Information Technology (21.8k)
  • Programming (13.1k)
  • Political Science (10.6k)
  • Home Science (8.1k)
  • Psychology (4.4k)
  • Sociology (7.1k)
  • English (68.2k)
  • Hindi (30.8k)
  • Aptitude (23.7k)
  • Reasoning (14.8k)
  • Olympiad (535)
  • Skill Tips (91)
  • RBSE (49.1k)
  • General (74.0k)
  • MSBSHSE (1.8k)
  • Tamilnadu Board (59.3k)
  • Kerala Board (24.5k)
  • Send feedback
  • Privacy Policy
  • Terms of Use
  • Refund Policy

If NCERT website is not working properly in your device kindly clear your browser cache, cookies, temp files and retry click to know more.

  • Skip to main content |
  • Skip to navigation |
  • Screen Reader Access |
  • Text Size: A- | A | A+
  • Site View: A A

assignment 1 geography class 10

Textbooks PDF (I-XII)

cbsencertsolutions

CBSE NCERT Solutions

NCERT and CBSE Solutions for free

Class 10 Social Science Geography Assignments

We have provided below free printable Class 10 Social Science Geography Assignments for Download in PDF. The Assignments have been designed based on the latest NCERT Book for Class 10 Social Science Geography . These Assignments for Grade 10 Social Science Geography cover all important topics which can come in your standard 10 tests and examinations. Free printable Assignments for CBSE Class 10 Social Science Geography , school and class assignments, and practice test papers have been designed by our highly experienced class 10 faculty. You can free download CBSE NCERT printable Assignments for Social Science Geography Class 10 with solutions and answers. All Assignments and test sheets have been prepared by expert teachers as per the latest Syllabus in Social Science Geography Class 10. Students can click on the links below and download all Pdf Assignments for Social Science Geography class 10 for free. All latest Kendriya Vidyalaya Class 10 Social Science Geography Assignments with Answers and test papers are given below.

Social Science Geography Class 10 Assignments Pdf Download

We have provided below the biggest collection of free CBSE NCERT KVS Assignments for Class 10 Social Science Geography . Students and teachers can download and save all free Social Science Geography assignments in Pdf for grade 10th. Our expert faculty have covered Class 10 important questions and answers for Social Science Geography as per the latest syllabus for the current academic year. All test papers and question banks for Class 10 Social Science Geography and CBSE Assignments for Social Science Geography Class 10 will be really helpful for standard 10th students to prepare for the class tests and school examinations. Class 10th students can easily free download in Pdf all printable practice worksheets given below.

Topicwise Assignments for Class 10 Social Science Geography Download in Pdf

Class 10 Social Science Geography Assignments

Advantages of Class 10 Social Science Geography Assignments

  • As we have the best and largest collection of Social Science Geography assignments for Grade 10, you will be able to easily get full list of solved important questions which can come in your examinations.
  • Students will be able to go through all important and critical topics given in your CBSE Social Science Geography textbooks for Class 10 .
  • All Social Science Geography assignments for Class 10 have been designed with answers. Students should solve them yourself and then compare with the solutions provided by us.
  • Class 10 Students studying in per CBSE, NCERT and KVS schools will be able to free download all Social Science Geography chapter wise worksheets and assignments for free in Pdf
  • Class 10 Social Science Geography question bank will help to improve subject understanding which will help to get better rank in exams

Frequently Asked Questions by Class 10 Social Science Geography students

At https://www.cbsencertsolutions.com, we have provided the biggest database of free assignments for Social Science Geography Class 10 which you can download in Pdf

We provide here Standard 10 Social Science Geography chapter-wise assignments which can be easily downloaded in Pdf format for free.

You can click on the links above and get assignments for Social Science Geography in Grade 10, all topic-wise question banks with solutions have been provided here. You can click on the links to download in Pdf.

We have provided here topic-wise Social Science Geography Grade 10 question banks, revision notes and questions for all difficult topics, and other study material.

We have provided the best collection of question bank and practice tests for Class 10 for all subjects. You can download them all and use them offline without the internet.

Related Posts

Class 10 Mathematics Areas related to Circles Assignments

Class 10 Mathematics Areas related to Circles Assignments

Class 10 Tamil Assignments

Class 10 Tamil Assignments

Class 10 French Assignments

Class 10 French Assignments

  • Class 6 Maths
  • Class 6 Science
  • Class 6 Social Science
  • Class 6 English
  • Class 7 Maths
  • Class 7 Science
  • Class 7 Social Science
  • Class 7 English
  • Class 8 Maths
  • Class 8 Science
  • Class 8 Social Science
  • Class 8 English
  • Class 9 Maths
  • Class 9 Science
  • Class 9 Social Science
  • Class 9 English
  • Class 10 Maths
  • Class 10 Science
  • Class 10 Social Science
  • Class 10 English
  • Class 11 Maths
  • Class 11 Computer Science (Python)
  • Class 11 English
  • Class 12 Maths
  • Class 12 English
  • Class 12 Economics
  • Class 12 Accountancy
  • Class 12 Physics
  • Class 12 Chemistry
  • Class 12 Biology
  • Class 12 Computer Science (Python)
  • Class 12 Physical Education
  • GST and Accounting Course
  • Excel Course
  • Tally Course
  • Finance and CMA Data Course
  • Payroll Course

Interesting

  • Learn English
  • Learn Excel
  • Learn Tally
  • Learn GST (Goods and Services Tax)
  • Learn Accounting and Finance
  • GST Tax Invoice Format
  • Accounts Tax Practical
  • Tally Ledger List
  • GSTR 2A - JSON to Excel

Are you in school ? Do you love Teachoo?

We would love to talk to you! Please fill this form so that we can contact you

You are learning...

Click on any of the links below to start learning from Teachoo ...

Geography is the study of the planet Earth and the various life sustaining elements that are found on it. You will study about this subject in every class of your secondary school life as it is one of the most important subjects to know about the place you are living in.

In Class X - Contemporary India - you will learn about various kinds of resources that are found on our planet. These resources are also known as the natural resources as they exist naturally and humans are not responsible for their existence.

You will start your journey by learning about the forest and wildlife resources which will teach you about the flora and fauna of our planet. 

After forest, you will move to learning about Water which is one of the most important natural resources for the existence of life on the planet.

At last, you will learn about various minerals and other energy resources that are necessary for the operations of various industries and how these manufacturing industries act as the lifelines of our economy.

At Teachoo, you will find everything related to the subject which includes concepts , NCERT questions, previous year questions as well as extra questions that we like to call Teachoo questions , which are specially curated for you by our experts .

Start your Teachoo journey by clicking below!

Happy Learning !

Chapter 1 Class 10 Geography - Resources and Development

Chapter 2 class 10 geography - forest and wildlife resources, chapter 3 class 10 geography - water resources, chapter 4 class 10 geography - agriculture, chapter 5 class 10 geography - minerals and energy resources, chapter 6 class 10 geography - manufacturing industries, chapter 7 class 10 geography - lifelines of national economy.

What's in it?

Hi, it looks like you're using AdBlock :(

Please login to view more pages. it's free :), solve all your doubts with teachoo black.

NCERT Books

NCERT-Geography-Book-Class-10

NCERT Geography Book Class 10 PDF Download [2020 -21 Edition Revised Syllabus]

NCERT Class 10 Geography Books : The National Council of Educational Research and Training (NCERT) publishes Geography textbooks for Class 10. The NCERT Class 10th Geography textbooks are well known for it’s updated and thoroughly revised syllabus. The NCERT Geography Books are based on the latest exam pattern and CBSE syllabus. NCERT has a good image when it comes to publishing the study materials for the students. NCERT keeps on updating the Geography books with the help of the latest question papers of each year. The Class 10 Geography books of NCERT are very well known for its presentation. The use of these books is not only suitable for studying the regular syllabus of various boards but it can also be useful for the candidates appearing for various competitive exams UPSC, Staff Selection Commission, PSU’s, Railways, PSC Exams.

NCERT Class 10 Geography Books PDF Download in English and Hindi Medium

NCERT Class 10 Geography Books are provided in PDF form so that students can access it at anytime anywhere. Class 10 NCERT Geography Books are created by the best professors who are experts in Geography and have good knowledge in the subject.

Contemporary India – II NCERT Geography Book Class 10 PDF in English Medium

  • Class 10 Geography NCERT Book in English Medium
  • Class 10 Geography NCERT Book in Hindi Medium

Geography Book Class 10:

Geography (Contemporary India – II)

  • Chapter 1 Resources and Development
  • Chapter 2 Forest and Wildlife Resources
  • Chapter 3 Water Resources
  • Chapter 4 Agriculture
  • Chapter 5 Minerals and Energy Resources
  • Chapter 6 Manufacturing Industries
  • Chapter 7 Lifelines of National Economy

Class 10 NCERT Geography Book PDF Download

भूगोल (समकालीन भारत – II)

  • Chapter 1 संसाधन और विकास
  • Chapter 2 वन और वन्यजीव संसाधन
  • Chapter 3 जल संसाधन
  • Chapter 4 कृषि
  • Chapter 5 खनिज और ऊर्जा संसाधन
  • Chapter 6 विनिर्माण उद्योग
  • Chapter 7 राष्ट्रीय अर्थव्यवस्था की जीवन रेखाएँ

Class 10 NCERT Geography Book in Hindi PDF

The NCERT syllabus mainly focuses on this book to make it student-friendly to make it useful for both the students and the competitive exam aspirants. The book covers a detailed Geography based on the syllabuses of various boards. NCERT Geography Books for Class 10 is perfectly compatible with almost every Indian education state and central boards.

We hope that this detailed article on NCERT Class 10 Geography Books helps you in your preparation and you crack the Class 10 exams or competitive exams with excellent scores. For your convenience, you can download PDFs of books and structure your study plan ahead. You should focus more on practicing previous year question papers too as this will further assist you in understanding the frequency of questions.

Leave a Comment Cancel reply

You must be logged in to post a comment.

  • NCERT Solutions
  • NCERT Solutions for Class 10
  • NCERT Solutions for Class 10 Social Science
  • Class 10 Geography

NCERT Solutions for Class 10 Geography

Download chapter-wise ncert solutions for class 10 geography.

NCERT Solutions for Class 10 Geography are important resources for board exam preparation. For most students, scoring marks in Geography is easy, as 70 to 80% of the questions appearing in exams are asked directly from NCERT Class 10 Geography textbook. So, to help them prepare better for the exam, we have provided NCERT Solutions for Class 10  Geography.

Students can have a look at the chapter-wise solutions of Geography – Contemporary India II from the table below. These NCERT Solutions provide the answers to all exercise questions. These answers will provide in-depth knowledge of each chapter and important topics. To access these solutions, click the links below.

Overview of CBSE Class 10 Geography Chapters

There are a total of 7 chapters in Class 10 Geography book. Here, we have provided a brief description of each chapter.

Chapter 1: Resources and Development

Geography Chapter 1 starts with the definition of resources. The classification of resources is explained based on the origin, exhaustibility, ownership, and potential of development. Further, the chapter talks about the development of resources and resource planning. This chapter mainly focuses on land resources by describing land utilisation, land use patterns in India, land degradation and conservation measures, soil as a resource and the classification of soil in-depth.

Topics Covered in Class 10 Geography Chapter 1 Resources and Development 

  • Development of Resources
  • Resource Planning – Resource Planning in India, Conservation of Resources
  • Land Resources
  • Land Utilisation
  • Land Use Pattern in India
  • Land Degradation and Conservation Measures
  • Soil as a Resource – Classification of Soils, Soil Erosion and Soil Conservation (excluding Box Information on State of India’s Environment)

List of Map Items in  C lass 10 Geography Chapter 1 Resources and Development 

a. Major soil Types

Chapter 2: Forest and Wildlife Resources

This chapter helps in understanding the different variety of flora and fauna that exists in nature and the reason behind the depletion of flora and fauna. The chapter further describes the conservation and distribution of forests and wildlife in India. In the end, students will get to know about some of the steps taken by common people to conserve the forests and wildlife resources.

Topics Covered in Class 10 Geography Chapter 2 Forest and Wildlife Resources

  • Conservation of Forest and Wildlife in India
  • Types and Distribution of Forests and Wildlife Resources
  • Community and Conservation

Chapter 3: Water Resources

The chapter starts by describing the availability of fresh water on the earth’s surface. Further, it explains the causes of water scarcity and the need for conserving and managing water resources. Students will learn about multi-purpose river projects and integrated water resources management. In the end, rain water harvesting and different methods of adopted rain water harvesting in different areas are explained in-depth.

Topics Covered in Class 10 Geography Chapter 3 Water Resources

  • Water Scarcity and the Need for Water Conservation and Management
  • Multi-Purpose River Projects and Integrated Water Resources Management
  • Rainwater Harvesting

List of Map Items in  C lass 10 Geography Chapter 3 Water Resources 

Dams: a. Salal b. Bhakra Nangal c. Tehri d. Rana Pratap Sagar e. Sardar Sarovar f. Hirakud g. Nagarjuna Sagar h. Tungabhadra

Chapter 4: Agriculture

India is primarily an Agriculture country. Farming has been practised in India for ages. This chapter is included in the textbook to help students understand the type of farming system practised in India, cropping pattern and major crops produced in India.

Topics Covered in Class 10 Geography Chapter 4 Agriculture 

  • Types of Farming – Primitive Subsistence, Intensive Subsistence, Commercial
  • Cropping Pattern – Major Crops, Food Crops Other Than Grains, Non Food Crops, Technological and Institutional Reforms
  • Food Security (excluding the impact of globalisation on agriculture)

List of Map Items in  C lass 10 Geography Chapter 4 Agriculture 

a. Major Areas of Rice and Wheat b. Largest/Major Producer States of Sugarcane, Tea, Coffee, Rubber, Cotton and Jute

Chapter 5: Minerals and Energy Resources

The earth’s crust is made up of different minerals. This chapter will explain Minerals and the occurrence of Minerals. The chapter further describes the classification of Minerals. Under the classification of minerals, each type is described in detail. In the end, the chapter briefs about conventional and non-conventional energy resources.

Topics Covered in Class 10 Geography Chapter 5 Minerals and Energy Resources 

  • What Is a Mineral?
  • Mode of Occurrence of Minerals – Where Are These Minerals Found?, Ferrous Minerals, Non-Ferrous Minerals, Nonmetallic Minerals, Rock Minerals
  • Conservation of Minerals
  • Energy Resources – Conventional Sources of Energy, Non-Conventional Sources of Energy
  • Conservation of Energy Resources

List of Map Items in  C lass 10 Geography Chapter 5 Minerals and Energy Resources 

Minerals – (Identification only) a. Iron Ore mine

b. Coal Mines

c. Oil Fields

  • Mumbai High

Power Plants (Locating and Labelling only)

Chapter 6: Manufacturing Industries

The manufacturing sector is considered the backbone of development. This chapter is especially included in the NCERT Geography book to help students understand the importance of manufacturing and the contribution of Industries to the National Economy. They will know the factors that impact the industrial location. Moreover, this chapter explains Ago-based Industries, Mineral-based industries, Chemical Industries, etc. In the end, the chapter explains how industrial pollution is degrading the environment and the major steps taken to prevent its degradation.

Topics Covered in Class 10 Geography Chapter 6 Manufacturing Industries 

  • Importance of Manufacturing – Industrial Location (excluding Industry Market Linkage), Agro-based Industry (excluding Cotton Textiles, Jute Textiles, Sugar Industry), Mineral based Industries (excluding Iron Steel Industry, Cement Industry), Industrial Pollution and Environmental Degradation, Control of Environmental Degradation

List of Map Items in  C lass 10 Geography Chapter 6 Manufacturing Industries 

Manufacturing Industries (Locating and Labelling Only) Software Technology Parks a. Noida b. Gandhinagar c. Mumbai d. Pune e. Hyderabad f. Bengaluru g. Chennai h. Thiruvananthapuram

Chapter 7: Lifelines of National Economy

The modern means of transport and communication serve as lifelines of the nation and its Economy. This chapter will help students understand how roadways, railways, pipelines, waterways, seaports, and airways have linked humans to the world and have contributed significantly to the growth of the Indian Economy. The chapter ends by describing the means of communication in India, International Trade and Tourism as Trade.

Topics Covered in Class 10 Geography Chapter 7 Lifelines of National Economy 

  • Major Seaports
  • Communication
  • International Trade
  • Tourism as a Trade

List of Map Items in  C lass 10 Geography Chapter 7 Lifelines of National Economy 

Major Ports: (Locating and Labelling) a. Kandla b. Mumbai c. Marmagao d. New Mangalore e. Kochi f. Tuticorin g. Chennai h. Vishakhapatnam i. Paradip j. Haldia International Airports: a. Amritsar (Raja Sansi – Sri Guru Ram Dass jee) b. Delhi (Indira Gandhi) c. Mumbai (Chhatrapati Shivaji) d. Chennai (Meenam Bakkam) e. Kolkata (Netaji Subhash Chandra Bose) f. Hyderabad (Rajiv Gandhi)

We hope students have found these NCERT Solutions for Class 10 Geography helpful in their studies. Keep learning and stay tuned for further updates on CBSE and other competitive exams. Download BYJU’S – The Learning App and subscribe to the YouTube channel to get interactive Maths and Science videos.

Also, explore – 

Frequently Asked Questions on NCERT Solutions for Class 10 Geography

Can the ncert solutions for class 10 geography be accessed free of cost, can i use the ncert solutions for class 10 geography to get my doubts cleared, how many chapters are present in the ncert solutions for class 10 geography, for chapter revision, are the ncert solutions for class 10 geography the best study resource, leave a comment cancel reply.

Your Mobile number and Email id will not be published. Required fields are marked *

Request OTP on Voice Call

Post My Comment

assignment 1 geography class 10

Register with BYJU'S & Download Free PDFs

Register with byju's & watch live videos.

Question and Answer forum for K12 Students

Class 10 Geography Chapter 1 Extra Questions and Answers Resource and Development

Class 10 Geography Chapter 1 Extra Questions and Answers Resource and Development

In this page, you can find CBSE Class 10 Geography Chapter 1 Extra Questions and Answers Resource and Development Pdf free download, NCERT Extra Questions for Class 10 Social Science  will make your practice complete.

Resource and Development World Class 10 Extra Questions and Answer Geography Chapter 1 Very Short Answers Type

Class 10 Geography Chapter 1 Extra Questions

Question 1. Mention two factors on which resource development depends. Answer:

  • Latest technology
  • Quality of humans as resource

Extra Questions and Answers Resource and Development

Question 2. What is resource? Give one example. Answer: Everything available in our environment which can be used to satisfy our needs, and which is technologically accessible, economically feasible and culturally acceptable can be termed as ‘resource’. For example, water is a resource.

Questions and Answers Resource and Development

Question 3. How can you say that resources are not free gifts of nature? Answer: Resources are, in fact, a function of human activities. Human beings, who themselves are essential components of resources, transform materials available in our environment into resources and use them. So, it is wrong to say that resources are free gifts of nature.

Question 4. What is meant by sustainable development? Answer: Sustainable development means ‘development should take place without damaging the environment, and development in the present should not compromise with the needs of the future generations.

Question 5. Why was Rio de Janeiro Earth Summit convened in 1992? Answer: The Summit was convened for addressing urgent problems of environmental protection and socio-economic development at the global level.

Question 6. What is Net Sown Area (NSA)? Answer: Area sown more than once in an agricultural year is known as Net Sown Area.

Question 7. What is meant by gross cropped area? Answer: Area sown more than once in an agricultural year plus net sown area is known as gross cropped area.

Question 8. What are the physical factors that determine the use of land? Answer: The physical factors that determine the use of land are — topography, climate and soil types.

Question 9. What are the human factors that determine the use of land? Answer: The human factors that determine the use of land are – population density, technological capability and culture and traditions etc.

Question 10. India has land under a variety of relief features. Name them. Answer: Mountains, plateaus, plains and islands.

Question 11. What are shelter belts? How have they proved helpful? Answer: Rows of trees which are planted in between the crops are called shelter belts. These shelter belts have proved helpful in the stabilisation of sand dunes and in stabilising the desert in western India.

Question 12. What do you mean by international resources? Answer: Resources that are owned and regulated by international institutions is called international resources. The oceanic resources beyond 200 nautical miles of the Exclusive Economic Zone belong to open ocean and no individual country can utilise these without the concurrence of international institutions.

Question 13. How does the contour farming help in the soil conservation? Answer: Ploughing along the contour lines can decelerate the flow of water down the slopes and thus helpful in the soil conservation.

Question 14. Name the four states where mining have caused severe land degradation. Answer: These states are- Jharkhand, Chhattisgarh, Madhya Pradesh and Odisha.

Question 15. What are the important factors in the formation of soil? Answer: The important factors in the formation of soil include relief, parent rock or bed rock, climate, vegetation and other forms of life and time.

Question 16. What is gully erosion? Answer: Gully erosion takes place when running water cuts deep ravines in the absence of vegetation. This type of erosion makes soil unfit for cultivation.

Question 17. Name the areas where red and yellow soils are found. Answer: The eastern and southern parts of the Deccan plateau, parts of Odisha, Chhattisgarh, southern parts of the middle Ganga plain and the piedmont zone of the Western Ghats.

Question 18. Where does the laterite soil develop? Answer: The laterite soil develops in areas with high temperature and heavy rainfall. This is the result of intense leaching due to heavy rainfall.

Question 19. What causes land degradation? Answer: Continuous use of land over a long period of time without taking appropriate measures to conserve and manage it causes land degradation.

Question 20. What is Agenda 21? Answer: Agenda 21 is a declaration signed by the world leaders in the Earth Summit held at Rio de Janeiro (Brazil) in 1992 in order to achieve global sustainable development.

Question 21. Which human factors have contributed to land degradation? Answer: Deforestation, overgrazing, mining and quarrying are some of the human factors which have contributed to land degradation.

Question 22. Name the states where overgrazing has caused land degradation. Answer: Gujarat, Rajasthan, Madhya Pradesh and Maharashtra.

Question 23. What are the necessary conditions for the development of any region? Answer: The availability of resources and corresponding changes in technology and institutions are the necessary conditions for the development of any region.

Question 24. How is over-irrigation responsible for land degradation? Answer: Over irrigation leads to waterlogging in the field which further leads to increase in salinity and alkalinity in the soil.

Question 25. What do you mean by bad land? Answer: Land that is highly unsuitable for cultivation is called bad land. Soil erosion is the factor which converts a fertile land into a bad land.

Resource and Development World Class 10 Extra Questions and Answer Geography Chapter 1 Short Answers Type

Question 1. Mention any two human activities which are responsible for the process of soil erosion. Explain the two types of soil erosion mostly observed in India. Answer: Large scale deforestation and mining are some of the human activities responsible for the process of soil erosion. The most common types of soil erosion in India are (a) Gully erosion, and (b) Sheet erosion.

(a) Gully erosion: It is the removal of soil along drainage lines by surface water run off. Once started, gullies will continue to move by head ward erosion or by slumping of the side walls unless steps are taken to stabilise the disturbance.

(b) Sheet erosion: Sometimes water flows as a sheet over large areas down a slope. In such cases the top soil is washed away. This is known as sheet erosion.

Question 2. Discuss the role of humans in resource development. (Imp) Answer: (i) Humans play an important role in resource development. They interact with nature through technology and create institutions to accelerate their economic development.

(ii) They convert materials available in our environment into resources. To fulfil their needs, they make the natural elements useful and valuable by dint of their intelligence, skill and technical knowledge.

(iii) For example, running water of rivers is a natural gift and it becomes a resource when man uses it (river water) for irrigation by constructing a canal. Man can also use this water for power generation by building dams on rivers. Thus, in the process of conversion of materials to resource creation, man’s role is more important.

Question 3. Classify resources on the basis of exhaustibility. OR Distinguish between renewable and non-renewable resources. Answer: On the basis of exhaustibility, resources are of two types: (i) Renewable resources, and (ii) Non-renewable resources. (i) Renewable resources: These resources can be renewed or reproduced by physical, chemical or mechanical processes. These are also known as replenishable resources. For example, solar and wind energy, water, forest and wildlife.

(ii) Non-renewable resources: These resources occur over a long geological time. For example, minerals and fossil fuels take millions of years in their formation. Some of the resources like metals are recyclable and some like fossil fuels cannot be recycled and get exhausted with their use.

Question 4. What is Agenda 21? What does it aim at? Answer: Agenda 21 is a non-binding, voluntarily implemented action plan of the United Nations with regard to sustainable development. It is a product of the Earth Summit i.e. UN Conference on Environment and Development (UNCED) which took place at Rio de Janeiro, Brazil, in 1992. The ‘21’ in Agenda 21 refers to the 21st century.

It aims at achieving global sustainable development. It is an agenda to combat environmental damage, poverty, disease through global co-operation on common interests, mutual needs and shared responsibilities. One major objective of Agenda 21 is that every local government should draw its own local agenda 21.

Question 5. Mention three factors that are involved in resource planning. OR ‘Resource planning is a complex process’. Explain. Answer: Resource planning is a complex process and it involves the following factors: (i) Identification and inventory of resources across the regions of the country. This involves survey¬ing, mapping and qualitative and quantitative estimation and measurement of the resources.

(ii) Evolving a planning structure endowed with appropriate technology, skill and institutional set up for implementing resource development plans.

(iii) Matching the resources development plans with overall national development plans.

Question 6. Mention any three characteristics of arid soils. Answer: The three characteristics of arid soils are- (i) These soils range from red to brown in colour. They are generally sandy in texture and saline in nature. In some areas, the salt content is very high and common salt is obtained by evaporating the water.

(ii) These soils are mainly found in Western Rajasthan. As the region is characterised by dry climate and high temperature, evaporation is faster. That is why soil lacks humus and moisture.

(iii) The lower horizons of the soil are occupied by kanker because of the increasing calcium content downwards. The kanker layer formations in the bottom horizons restrict the infiltration of water up. This makes the soil unsuitable for cultivation.

(iv) These soils can be converted into cultivable land by proper irrigation.

Question 7. Distinguish between khadar and bangar soils. (Imp) OR Classify alluvial soils on the basis of their age

(i) This soil belongs to the category of new alluvium. (i) This soil belongs to the category of old alluvium.
(ii) It contains fine soil particles and is light in colour. (ii) It contains coarse soil particles and is dark in colour.
(iii) It is very fertile because of having fine particles. (iii) It is comparatively not very fertile because of having kanker nodules with calcium carbonate.
(iv) It is found along the banks, flood plains and delta regions. (iv) It is found above the level of flood plains.

Question 8. Which is called regur soil? Mention any three characteristics of this soil. (Imp) Answer: Black soil is also called regur soil. This soil is ideal for growing cotton and is also known as black cotton soil. Three characteristic features of this soil are-

  • The black soils are made up of extremely fine (clayey) material. They are well-known for their capacity to hold moisture.
  • These soils are rich in soil nutrients, such as calcium carbonate, magnesium, potash and lime but are deficient in phosphoric contents.
  • They become sticky when wet and develop cracks on drying up. These cracks help in mixing air in the soil.

Question 9. How does red soil develop and in which part of India? What makes it look red and yellow? Or How is red soil formed? Mention its three characteristics. Answer: Red soil develops on crystalline igneous rocks in areas of low rainfall in the eastern and southern parts of the Deccan plateau. Three characteristics of red soil-

  • This soil develops a reddish colour due to diffusion of iron in crystalline and metamorphic rocks. It looks yellow when it occurs in a hydrated form.
  • It is porous in nature and more fertile. It is also very thick.
  • On the uplands, this soil consists of loose gravels and highly coarse materials, but in the lowlands it is rich, deep dark coloured, fertile and red.

Question 10. Which is the most widespread and important soil of India? Mention its important characteristics. Answer: Alluvial soil is found in the river deltas of the eastern coast. Three main features of this type of soil are –

  • This soil consists of various proportions of sand, silt and clay. It is a very fertile soil.
  • Mostly these soils contain adequate proportion of potash, phosphoric acid and lime which are ideal for the growth of sugarcane, paddy, wheat etc.
  • Due to its high fertility, regions of alluvial soils are intensively cultivated and densely populated.

Question 11. Name the soil type which is widely found in Western Rajasthan. Explain two important characteristics of this soil type which make it unsuitable for cultivation. (Imp) Answer: The three characteristics of arid soils are- (i) These soils range from red to brown in colour. They are generally sandy in texture and saline in nature. In some areas, the salt content is very high and common salt is obtained by evaporating the water.

Question 12. Explain the distribution of the black soil. Answer: This type of soil is typical of the Deccan trap region spread over northwest Deccan plateau and is made up of lava flows. They cover the plateaus of Maharashtra, Saurashtra, Malwa, Madhya Pradesh and Chhattisgarh and extend in the south east direction along the Godavari and the Krishna valleys.

Question 13. Describe the laterite soil under the following headings: (i) Why are they called laterite? (ii) Distribution of this soil (iii) Crops grown in these soils Answer: (i) The word ‘laterite’ has been derived from the Latin word ‘later’ which means brick. Since its colour is red and resembles brick colour that is why it is called laterite soil. (ii) Laterite soils are mainly found in Karnataka, Kerala, Tamil Nadu, Madhya Pradesh, and the hilly areas of Odisha and Assam. (iii) Crops grown in these soils are tea, coffee and cashew nuts.

Question 14. What do you mean by land degradation? What is the area of degraded land in India? Mention the name of various types of wasteland and their percentage share. Answer: Land degradation is a process in which the value of the biophysical environment is affected by a combination of human-included processes acting upon the land. At present, there are about 130 million hectares of degraded land in India. Approximately, 28 percent of it belongs to the category of forest degraded area, 56 percent of it is water eroded area, 6 percent of it is saline and alkaline land and 10 percent of it is wind eroded area.

Question 15. Where are forest soils found in India? Mention the major characteristics of these soils. Answer: Forest soils are found in the hilly and mountainous areas where sufficient rainforests are available. Major characteristics of these soils are:

  • The texture of these soil varies according to the mountain environment where they are formed. They are loamy and salty in valley sides and coarse grained in the upper slopes.
  • In the snow-covered areas of the Himalayas, these soils experience denudation and are acidic with low humus content.
  • The soils found in the lower parts of the valleys particularly on the river terraces and alluvial fans are fertile.

Resource and Development World Class 10 Extra Questions and Answer Geography Chapter 1 Long Answers Type

Question 1. Classify resources on the basis of ownership. Mention major features of these resources. OR Explain the classification of resources on the basis of ownership. (Imp) Answer: On the basis of ownership, there are four types of resources-

  • Individual resources
  • Community owned resources
  • National resources
  • International resources

Individual resources: These resources are owned privately by individuals. Many farmers own land which is allotted to them by government against the payment of revenue. People in urban areas also own plots, houses and other property. Other examples of resources owned by individu¬als include plantation, pasture lands, ponds, water in wells, etc.

Community owned resources: These resources are accessible to all the members of the commu¬nity. Village commons, public parks, picnic spots, playgrounds etc. are accessible to all the people living in that area.

National resources: All the resources such as water resources, forests, wildlife, land within the political boundaries and oceanic area upto 12 nautical miles from the coast termed as territorial water and resources therein belong to the nation and therefore, known as national resources.

International resources: There are international institutions which regulate some resources. The oceanic resources beyond 200 nautical miles of the Exclusive Economic Zone belong to open ocean and no individual country can utilise these resources without the concurrence of international institutions.

Question 2. Explain the classification of resources on the basis of the status of development. (Imp) OR Distinguish between stock resources and reserve resources. (Imp) Answer: On the basis of the status of development resources are classified in the following categories:

  • Potential resources
  • Developed resources

Potential resources: Potential resources are those resources which are found in a region, but have not been utilised. For example, the western parts of India particularly Rajasthan and Gujarat have huge potential for the development of wind and solar energy. But these have not been developed properly till date.

Developed resources: These resources are surveyed and their quality and quantity have been determined for utilisation. The development of resources depends on technology and level of their feasibility.

Stock: Materials in the environment which have the potential to satisfy human needs but they are not accessible due to lack of appropriate technology, are included among stock resources. For example, water is a compound of two inflammable gases hydrogen and oxygen, which can be used as a rich source of energy. But in absence of required technology to use them for this purpose, it is considered as stock resources.

Reserves: These are the subset of the stock, which can be put into use with the help of existing technology but their use has not been started. These can be used for meeting future requirements. River water can be used for generating hydroelectric power but presently, it is being utilised only to a limited extent. Thus, water in the dams, forests etc. is a reserve which can be utilised in the future.

Question 3. What is soil? What is its importance in human life? Explain the factors that contribute to soil formation. Answer: Soil is the uppermost layer of the unconsolidated particles found on the surface of the earth. It is made up of parent rocks and vegetation. It is rich in both organic and inorganic materials and supports the growth of plants.

Soil is the most important renewable natural resource. It is the medium of plant growth and supports different types of living organisms on the earth.

Soil is a living system. It takes millions of years to form soil upto a few cm in depth. There are different factors which help in formation of soil:

  • Relief, parent rock or bed rock, climate, vegetation and other forms of life and time are important factors in the formation of soil.
  • Various forces of nature such as change in temperature, actions of running water, wind and glaciers, activities of decomposers, etc. contribute to the formation of soil.
  • Chemical and organic changes which take place in the soil also contribute to soil formation.

Question 4. What is meant by soil profile? Draw a labelled diagram of soil profile. Answer: Soil profile is the sequence, colour, texture and nature of the horizons superimposed one above the other and exposed in a pit-section dug through the soil mantle.

Class 10 Geography Chapter 1 Extra Questions and Answers Resource and Development 2

The various layers of soil profile are:

  • Top soil or the upper soil layer
  • Sub soil weathered rocks sand and silt clay
  • Substratum weathered parent rock material
  • Unweathered parent bed rock

Top soil of the uppermost layer is the real soil. It contains humus and inorganic materials. Below it is the subsoil which consists of rocks. sand particles and clay. The third layer, which comes below the second layer is made up of weathered parent rock material and the fourth layer is made up of parent bed rock.

Question 5. What is soil erosion? What factors contribute to it? Mention various types of soil erosion. Answer: Soil erosion is the washing or blowing away (by wind or water) of the top layer of soil. This is a serious problem particularly for farmers. The process of soil formation and erosion, go on side by side. Generally, there is a balance between the two. But when this balance is disturbed, soil erosion takes place. The factors that contribute to soil erosion are-

  • Deforestation
  • Over grazing
  • Construction and mining activities
  • Natural forces such as wind, glacier and water.

There are different types of soil erosion-

Water erosion and surface water runoff: The loss of top soil due to water is called water erosion. During rainy season, when raindrops fall directly on top soil, they loosen the material binding it together. As a result, small fragments get detached. If the rainfall continues, water gathers on the ground, causing water flow on the land surface, known as surface water runoff. This runoff carries the detached soil away.

Sheet erosion: Sometimes water flows as a sheet over large areas down a slope. In such cases the top soil is washed away. This is known as ‘sheet erosion’.

Rill erosion: Sometimes rainfall does not soak into the soil, and gathers on the surface and runs downhill, forming small channels of water called rills. These rills get dry up after the rainfall but the stream bed created by the temporary stream becomes prominent.

Gully erosion: The running water cuts through the clayey soils and makes deep channels as gullies. This type of erosion poses problems for farmers because the affected land is not put for cultivation.

Wind erosion: Wind blows loose soil off flat or sloping land known as wind erosion.

Defective methods of farming: Due to defective farming methods, there arises the problem of soil erosion. Ploughing in a wrong way i.e. up and down the slope form channels for the quick flow of water leading to soil erosion.

Question 6. What is meant by land degradation? Write five human activities which are mainly responsible for land degradation in India. (Imp) Answer: Land degradation is a process through which the land becomes unsuitable for agricultural activities. The following human activities are responsible for land degradation:

(i) Mining: Mining sites are abandoned after excavation work is complete leaving deep scars and traces of over burdening. In states like Jharkhand, Chhattisgarh, Madhya Pradesh and Odisha deforestation due to mining have caused severe land degradation.

(ii) Overgrazing: Overgrazing occurs when plants are exposed to intensive grazing for extended periods of time, or without sufficient recovery periods. In states like Gujarat, Rajasthan, Madhya Pradesh and Maharashtra overgrazing is one of the main reasons for land degradation.

(iii) Over irrigation: In states of Punjab, Haryana and western Uttar Pradesh, over irrigation is responsible for land degradation. It occurs due to water logging that leads to increase in salinity and alkalinity in the soil.

(iv) Mineral processing: The mineral processing like grinding of limestone for cement industry and calcite and soapstone for ceramic industry generate huge quantity of dust in the atmosphere. It retards the process of infiltration of water into the soil after it settles down on the land.

(v) Industrial effluents as waste have also become a major source of land degradation in many parts of the country.

Resource and Development World Class 10 Extra Questions and Answer Geography Chapter 1 Higher Order Thinking Skills (HOTS) Questions

Question 1. Why is resource planning so important in a country like India? Give reasons. Answer: (i) India has enormous diversity in the availability of resources. There are regions which are rich in certain types of resources but are deficient in some other resources.

(ii) There are some regions which can be considered self-sufficient in terms of the availability of resources and there are some regions which have acute shortage of some vital resources.

(iii) The states of Jharkhand, Chhattisgarh and Madhya Pradesh are rich in minerals and coal deposits. Arunachal Pradesh has no dearth of water resources but lacks in infrastructrual development. The state of Rajasthan is gifted with solar and wind energy but lacks in water resources. Ladakh has rich cultural heritage but it is deficient in water; infrastructure etc. This calls for balanced resource planning at the national, state, regional and local levels.

Question 2. What are the various methods of conservation of soil? Explain. Answer: There are various methods of soil conservation:

  • Contour farming in mountainous regions: Ploughing along the contour lines can decelerate the flow of water down the slopes.
  • Terrace farming: Steps can be cut out on the slopes making terraces. Terrace cultivation restricts erosion.
  • Strip farming: Large fields can be divided into strips. Strips of grass are left to grow between the crops. This breaks up the force of the wind.
  • Shelter belts: Planting lines of trees to create shelters have contributed significantly to the stabilisation of sand dunes and in stabilising the desert in western India.
  • Other methods include afforestation, control on grazing, etc.

Question 3. Why is there a need to conserve resources? What was Gandhiji’s opinion regarding the conservation of resources? Answer: Resources are vital for any developmental activity. But irrational consumption and over-use of resources for selfish purposes may lead to socio-economic and environmental problems. In order to overcome these problems, resource conservation at various levels is important.

Gandhiji voiced his concern about resource conservation in these words, “There is enough for everybody’s need and not for anybody’s greed”. He placed the greedy and selfish individuals and exploitative nature of modern technology as the root cause for resource depletion at the global level. He didn’t believe in mass production and wanted to replace it with the production by the masses.

Question 4. ‘India has land under a variety of relief features’. Support the statement with the help of a pie chart. Answer: The name of these relief features are mountains, plateaus, plains and islands.

Class 10 Geography Chapter 1 Extra Questions and Answers Resource and Development 3

  • About 43 percent of the land area is plain which provides facilities for agricultural and industrial activities.
  • Mountains account for 30 percent of the total surface area of the country and ensure perennial flow of some rivers. They also provide facilities for tourism and ecological aspects.
  • About 27 percent of the area of the country is the plateau region. It possesses rich reserves of minerals, fossil fuels and forests.

Resource and Development World Class 10 Extra Questions and Answer Geography Chapter 1 Value-based Questions (VBQs)

Question 1. Natural resources are of great importance. Which values are associated with them? Answer: The values associated with resources are-

  • They are used to satisfy human needs. For example, water is a resource and it is used for drinking, irrigation, cleaning purposes and cooking.
  • Resources make human life simpler and happier. They bring quality change in man’s life.
  • Resources are the basic requirements for country’s economic development. Today the countries which have more resources are considered as developed and prosperous.
  • Land resources support natural vegetation, wildlife, human life as well as man’s economic activities.
  • Water is an important resource because we cannot survive without it.

Question 2. Mention three problems which have been caused due to indiscriminate use of resources. Answer:

  • Depletion of resources for satisfying the greed of few individuals.
  • Accumulation of resources in few hands, which, in turn, divided the society into two groups- haves and have nots (the rich and the poor).
  • Indiscriminate exploitation of resources has led to global ecological crises such as, global warming, ozone layer depletion, environmental pollution and land degradation.

Resource and Development World Class 10 Extra Questions and Answer Geography Chapter 1 Map-based Questions

Question 1. Identify six major soil types shown in the given map of India. Answer:

  • Forest and mountainous soil
  • Alluvial soil
  • Red and yellow soil
  • Laterite soil and

Class 10 Geography Chapter 1 Extra Questions and Answers Resource and Development 4

AssignmentsBag.com

AssignmentsBag.com

Assignments For Class 10 Social Science Geography

Assignments for Class 10 Social Science Geography have been developed for Standard 10 students based on the latest syllabus and textbooks applicable in CBSE, NCERT and KVS schools. Parents and students can download the full collection of class assignments for class 10 Social Science Geography from our website as we have provided all topic wise assignments free in PDF format which can be downloaded easily. Students are recommended to do these assignments daily by taking printouts and going through the questions and answers for Grade 10 Social Science Geography. You should try to do these test assignments on a daily basis so that you are able to understand the concepts and details of each chapter in your Social Science Geography book and get good marks in class 10 exams.

Assignments for Class 10 Social Science Geography as per CBSE NCERT pattern

All students studying in Grade 10 Social Science Geography should download the assignments provided here and use them for their daily routine practice. This will help them to get better grades in Social Science Geography exam for standard 10. We have made sure that all topics given in your textbook for Social Science Geography which is suggested in Class 10 have been covered ad we have made assignments and test papers for all topics which your teacher has been teaching in your class. All chapter wise assignments have been made by our teachers after full research of each important topic in the textbooks so that you have enough questions and their solutions to help them practice so that they are able to get full practice and understanding of all important topics. Our teachers at https://www.assignmentsbag.com have made sure that all test papers have been designed as per CBSE, NCERT and KVS syllabus and examination pattern. These question banks have been recommended in various schools and have supported many students to practice and further enhance their scores in school and have also assisted them to appear in other school level tests and examinations. Its easy to take print of thee assignments as all are available in PDF format.

Some advantages of Free Assignments for Class 10 Social Science Geography

  • Solving Assignments for Social Science Geography Class 10 helps to further enhance understanding of the topics given in your text book which will help you to get better marks
  • By solving one assignments given in your class by Social Science Geography teacher for class 10 will help you to keep in touch with the topic thus reducing dependence on last minute studies
  • You will be able to understand the type of questions which are expected in your Social Science Geography class test
  • You will be able to revise all topics given in the ebook for Class 10 Social Science Geography as all questions have been provided in the question banks
  • NCERT Class 10 Social Science Geography Workbooks will surely help you to make your concepts stronger and better than anyone else in your class.
  • Parents will be able to take print out of the assignments and give to their child easily.

All free Printable practice assignments are in PDF single lick download format and have been prepared by Class 10 Social Science Geography teachers after full study of all topics which have been given in each chapter so that the students are able to take complete benefit from the worksheets. The Chapter wise question bank and revision assignments can be accessed free and anywhere. Go ahead and click on the links above to download free CBSE Class 10 Social Science Geography Assignments PDF.

Assignments For Class 10 Social Science Geography

You can download free assignments for class 10 Social Science Geography from https://www.assignmentsbag.com

You can get free PDF downloadable assignments for Grade 10 Social Science Geography from our website which has been developed by teachers after doing extensive research in each topic.

On our website we have provided assignments for all subjects in Grade 10, all topic wise test sheets have been provided in a logical manner so that you can scroll through the topics and download the worksheet that you want.

You can easily get question banks, topic wise notes and questions and other useful study material from https://www.assignmentsbag.com without any charge

Yes all test papers for Social Science Geography Class 10 are available for free, no charge has been put so that the students can benefit from it. And offcourse all is available for download in PDF format and with a single click you can download all assignments.

https://www.assignmentsbag.com is the best portal to download all assignments for all classes without any charges.

Related Posts

Assignments For Class 11 Mathematics Complex Numbers And Quadratic Equation

Assignments For Class 11 Mathematics Complex Numbers And Quadratic Equation

Assignments For Class 8 Computer Science

Assignments For Class 8 Computer Science

Assignments For Class 4 French

Assignments For Class 4 French

IMAGES

  1. NCERT Solutions Class 10 Geography Chapter 1 Resources and Development

    assignment 1 geography class 10

  2. class 10 geography chapter 1| #Part-2

    assignment 1 geography class 10

  3. Geography assignment-1 Lab manual social science(s.s.t) class-10 all solution Cbse extra learning

    assignment 1 geography class 10

  4. SOLUTION: Chapter 1 geography class 10

    assignment 1 geography class 10

  5. NCERT Class

    assignment 1 geography class 10

  6. Shortnotes Class 10 Geography

    assignment 1 geography class 10

VIDEO

  1. Manav Bhugol

  2. 🔥class 10 geography chapter 1||geography class 10 chapter 1 bihar board

  3. Class 10 Geography Chapter-1 Resources and Development Important Question Answer for Board Exam

  4. Class 10 Geography Chapter 1 Resource and development -Explanation

  5. Resources and Development

  6. Resources and Development Geography Class 10 Notes #class10socialscience #shorts #trending

COMMENTS

  1. NCERT Solutions For Class 10 Geography Social Science Chapter 1

    NCERT Solutions For Class 10 Geography Social Science ...

  2. NCERT Solutions for Class 10 Geography Chapter 1 Resources and

    15th January 2024. NCERT Solutions for Class 10 Social Science Geography Chapter 1 Resources and Development help students to score good marks in the exams. These NCERT Solutions are prepared by expert teachers and based on the latest pattern and edition of NCERT book. Here we have provided answers to all the questions in a very easy language.

  3. NCERT Solutions Class 10 Geography Chapter 1 Resources and Development

    on September 3, 2023, 4:25 AM. NCERT Solutions for Class 10 Geography Chapter 1 Resources and Development in English and Hindi Medium updated for new academic session 2024-25 based on current CBSE Curriculum and new NCERT Books for 2024-25. Ask your doubts and reply to the questions asked by other users through Discussion Forum of Tiwari Academy.

  4. CBSE Notes Class 10 Geography Chapter 1

    CBSE Notes Class 10 Geography Chapter 1

  5. NCERT Solutions Class 10 Geography Chapter 1 Resources and ...

    Vedantu's Class 10 Geography NCERT Solutions are your life raft! Aligned with the latest CBSE Class 10 Social Science syllabus, the Class 10 Geography Chapter 1 offers a comprehensive guide to understanding this pivotal era.Dive deep into sustainable development, resource planning, land resources, and more, explaining clearly and breaking down complex terms, making geography accessible and ...

  6. NCERT Solutions for Class 10 Geography Chapter 1 Free PDF Download

    Sub-topics covered under NCERT Solutions for Class 10 Geography Chapter 1. 1.1 Types of Resources- The topic discusses the various types of resources which mention below: 1.1.1 On the basis of Origin. Biotic Resources-like flora and fauna. Abiotic Resources- like rocks and metals. 1.1.2 On the basis of Exhaustibility.

  7. NCERT Solutions For Class 10 Social Science Geography Chapter 1

    April 7, 2024. Table of Contents. NCERT Solutions for Class 10 Social Science Geography Chapter 1: Students can use NCERT Solutions Class 10 Geography Chapter 1 Resources and Development as a valuable study tool to help them get ready for the CBSE exams. Students will learn the answer writing style by reading through these solutions, which will ...

  8. CBSE Notes Class 10 Geography Chapter 1- Resources and Development

    Resources and Development is the first chapter in CBSE Class 10 Geography, which discusses the concept of resources and their various forms. The chapter covers the different types of resources such as natural, human-made, and human resources, and their utilization for the development of society. It also discusses the process of resource ...

  9. NCERT Solutions for Class 10th: Ch 1 Resources and Development Geography

    4. Solve the puzzle by following your search horizontally and vertically to find the hidden answers. (i) Natural endowments in the form of land, water, vegetation and minerals. (ii) A type of non-renewable resource. (iii) Soil with high water retaining capacity. (iv) Intensively leached soils of the monsoon climate.

  10. NCERT Solutions Class 10 Social Science Geography Chapter 1 Resources

    In these NCERT Solutions for Class 10 Social Science, we have discussed all types of NCERT intext questions, exercise questions as well as multiple choice type questions. Concepts covered in Class 10 Social Science - Geography chapter 1 Resources and Development are : Resources and Development are Changing Land-use Pattern

  11. Textbooks PDF (I-XII)

    NCERT, Sri Aurobindo Marg, New Delhi-110016. [email protected]. +91 8800440559

  12. Class 10 Social Science Geography Assignments

    Class 10 Students studying in per CBSE, NCERT and KVS schools will be able to free download all Social Science Geography chapter wise worksheets and assignments for free in Pdf. Class 10 Social Science Geography question bank will help to improve subject understanding which will help to get better rank in exams.

  13. Geography Class 10

    Chapter 5 Class 10 Geography - Minerals and Energy Resources. Concepts. MCQ Questions (1 Mark) Assertion Reasoning. Picture Based Questions (MCQ) Fill in the blanks (MCQ) Map Based Questions. Past Year Questions - 3 Marks. Past Year Questions - 5 Marks.

  14. NCERT Geography Book Class 10 PDF Download [2020 -21 Edition Revised

    Class 10 NCERT Geography Book PDF Download. भूगोल (समकालीन भारत - II) Chapter 1 संसाधन और विकास. Chapter 2 वन और वन्यजीव संसाधन. Chapter 3 जल संसाधन. Chapter 4 कृषि. Chapter 5 खनिज और ऊर्जा ...

  15. NCERT Solutions for Class 10 Geography

    NCERT Solutions for Class 10 Geography

  16. Class 10 CHAPTER 1 GEOGRAPHY Resources and Development

    Class 10 CHAPTER 1 GEOGRAPHY Resources and Development - Free download as PDF File (.pdf), Text File (.txt) or read online for free.

  17. Geography Assignment Class 10 Chapter

    CBSE Class 10 Geography - Resources and Development (1) - Free download as PDF File (.pdf), Text File (.txt) or read online for free.

  18. GEOGRAPHY ASSIGNMENT Class 10 Chapter 1

    GEOGRAPHY ASSIGNMENT class 10 chapter 1 - Free download as Word Doc (.doc / .docx), PDF File (.pdf), Text File (.txt) or read online for free. This document contains a geography assignment on resources and development with questions covering various topics: 1. One mark questions define key terms like contour ploughing, waste land, and sustainable development and ask students to identify ...

  19. Class 10 Geography Chapter 1 Extra Questions and Answers Resource and

    Resource and Development World Class 10 Extra Questions and Answer Geography Chapter 1 Very Short Answers Type. Class 10 Geography Chapter 1 Extra Questions. Question 1. Mention two factors on which resource development depends. Answer: Latest technology. Quality of humans as resource. Question 2.

  20. Assignments For Class 10 Geography

    https://www.assignmentsbag.com is the best portal to download all assignments for all classes without any charges. Assignments for Class 10 Social Science Geography have been developed for Standard 10 students based on the latest syllabus and textbooks applicable in.

  21. SOCIAL SCIENCE

    FOR STUDY NOTES AND TRICKS JOIN SSLC WHATSAPP CHANNEL : https://whatsapp.com/channel/0029VaX00UQF6smxeuAFod0pSSLC COMMUNITY GROUP LINK : https://chat.whatsap...